You are on page 1of 92

KAPLAN LSAT PREP

LSAT

RELEASED TEST II
EXPLAINED

A Guide to the October, 1991 LSAT

KAPLAN
The answer to the test question.
1996 Stanley H. Kaplan Educational Center Ltd

All rights reserved. No part of this book may be reproduced in any form, by
photostat, microfilm, xerography or any other means, or incorporated into any
information retrieval system, electronic or mechanical, without the written permission
of Stanley H. Kaplan Educational Center Ltd.
SECTION I:
READING COMPREHENSION

© K A PL A N 1
LSAT PREP _________________________________________________________________ LSAT Test II Explained: Section I

Passage 1—Langston Hughes


(Q. 1-6)
Topic and Scope: Langston Hughes’s poetry; specifically, the relationship of his poems to
folk traditions that preceded him and the African American literature that followed him.

Purpose and Main Idea: The author argues that Langston Hughes’s approach to poetry
helped to change the nature of African American literature. His main idea is that Hughes’s
poetry established the groundbreaking principle that African American literature needn’t
conform to European standards in terms of style and substance.

Paragraph Structure: ¶1 briefly outlines how Hughes’s poetry broke from the accepted
conventions of African American literature, and then goes into a long description of those
conventions. This ¶ really boils down to the idea that Hughes, in contrast to his
predecessors, rebelled against “Europeanizing” his poetry.

¶2 describes the style and substance of Hughes's poetry; seemingly effortless, but actually
carefully wrought. Essentially, believes the author, Hughes's work is folk literature that
incorporates the styles and themes of Black folk culture. Hughes's poetry, this ¶ goes on to
add, helped to “broaden the linguistic and thematic range” of African American literature.

The Big Picture:

• A passage like this one—which opens with a mass of seemingly disconnected


details, obscuring its scope and purpose—may not be a great place to begin your
Reading Comprehension work on Test Day. You should begin the section by
locating and working on passages in which scope and purpose are both readily
apparent early on.

• Like many LSAT passages, this one contains a lot of details. Don’t stop to
contemplate and absorb the details as you read through the passage. Instead, just
note where they appear, so that you can come back to them quickly if you need to.
Remember, you get points for answering questions correctly, not for memorizing or
dwelling on passage details. (For instance, see Q. 1, below.)

2 © K A PL A N
LSAT PREP _________________________________________________________________ LSAT Test II Explained: Section I

The Questions:

1. (C)
Lines 37-40 list “naming and enumeration” as one way in which Black folk culture
influenced Hughes's poetry. (“Hyperbole and understatement” or “street-talk rhyming”
could just as easily have been chosen for the right answer.)

(A) Kind of a gibberish answer. That Hughes considered his poetry “artless” doesn’t mean
that it’s “ambiguous.” Meanwhile, what the author thinks is “deceptive” is not Hughes's
“meanings,” but the way in which the poet’s work seems so impromptu and yet is so
carefully wrought. The poems fool one—they seem lighter than they really are.

(B) The care and craft Hughes used in composing his poems is “common to all folk
literature” (line 49), and not a consequence of the influence Black folk culture exercised on
him.

(D) That the poetry contains “monologues” and “narrative compression” doesn’t mean
that it must have been written as a narrative in the first person (which isn’t identified as a
Black folk trait anyway).

(E) That Black folk culture included religious themes doesn’t mean that Hughes's poetry
necessarily reflects them.

• Here’s a perfect example of what we discussed just prior to Q. 1. Memorizing details


as you read is unnecessary; what is necessary is an awareness of where a question’s
answer is likely to be yielded up. If you spotted the first part of ¶2 as the place where
the author describes Black folk influences on Hughes, it is there and nowhere else
you will seek your answer to Q. 1; and once you’ve reread or even just skimmed
there, (C) jumps out.

2. (B)
Lines 43-46 say that Hughes considered his poetry to be “impromptu and
impressionistic”— not carefully crafted. Lines 46-48, as signaled by the Keyword “Yet,”
indicate that the author holds an entirely different opinion of Hughes’s poetry. In other
words, we learn after the fact what the phrase “deceptive veil” refers to: namely, the
author’s belief that contrary to its artless appearance, Hughes’s poetry was actually
carefully wrought.

(A) By “oral techniques” (A) probably means lines 38-42, but those lines only precede the
“deceptive veil” reference; they don’t lead to it. Line 42 takes off on a new idea, which is
explained immediately thereafter.

(C) What is meant by the phrase “representative details”? This choice, even more than (A),
is far too vague to endorse.

(D) Au contraire. The passage explicitly states that Hughes’s rejected European literary
methods.

© K A PL A N 3
LSAT PREP _________________________________________________________________ LSAT Test II Explained: Section I

(E) The passage doesn’t reveal what specific issues Hughes addressed in his poetry. Maybe
they were social, political, and aesthetic. (Choice (E) probably hearkens way back to lines 6-
7, which is too far away from line 42 to be relevant. Besides, that Weary Blues failed to
promote racial harmony doesn’t mean that Hughes's work in general was socially and
politically disengaged.)

• The correct answer to a question that provides a line reference will usually be found
in the lines around the reference itself. Be sure to read the lines before (and
especially after) the reference closely before you settle on any choice.

• Be an ever-vigilant critical reader. When the author alleges something like “There is a
deceptive veil of artlessness,” don’t assume that it’s up to you to decide what that
means. Demand that the author explain himself! If you seek an explanation for a
baffling reference, one will usually be found—and will almost surely lead to a
question’s correct answer.

• Whenever you see a familiar Keyword, a red flag should go up in your mind. An
important point is sure to follow.

3. (D)
“Even [slave] spirituals...had been Europeanized” (lines 14-19) to make them acceptable to
broader American society. How so? Keep reading. During the Civil War the songs were
praised for their uniqueness, but by 1916 Jubilee Songs of the United States had been adapted
to appeal to broader American tastes and became a specific example of the
“Europeanizing” trend.

(A) No. If anything, the author would say that Jubilee is simply one work in a long line of
Black writings that were designed to encourage White acceptance of Blacks.

(B) The author draws no connection between Jubilee and The Weary Blues. Indeed, the
author says that The Weary Blues broke past Black literary traditions. Furthermore, the
author never states that African American art was “fashionable”; the word is used to
describe the church where Burleigh sang.

(C) Since the author says that Jubilee was adapted to appeal to broader American tastes, he
obviously wouldn’t argue that it authentically replicates African American folk songs.

(E) There’s no basis for this analogy because the passage doesn’t mention the contents of
The Weary Blues.

• The correct answer to an inference question will never stray very far from the spirit
of the text. If you have to go through a lot of mental gymnastics to justify a choice,
you’re probably trying to justify the wrong answer.

• It is the purpose of sentences like lines 23-29 that LSAT questions are set up to test. If
you see that this sentence’s purpose is to serve as support for lines 14-19, you get Q. 3
right quickly. If not, you struggle for a long time and may even get it wrong.

4 © K A PL A N
LSAT PREP _________________________________________________________________ LSAT Test II Explained: Section I

4. (B)
In lines 14-19, the author points out that Blacks were forced to “Europeanize” their “sorrow
songs” after the Civil War to make them more acceptable to American society. Blacks were
forced to do so even though White writers had earlier been favorably impressed with “non-
Europeanized” versions of these songs. In other words, the author mentions the reaction of
White writers to non-Europeanized versions of these songs in order to contrast pre- and
post-war attitudes toward them.

(A) is beyond the scope of the passage, which doesn’t address the issue of Northern vs.
Southern attitudes.

(C) According to the author, Blacks Europeanized their literature in order to make it more
acceptable to White society—which is hardly “unrelated” to Whites’ “standards.”

(D) is beyond the scope of the passage as well. The author never compares the qualities of
Europeanized and non-Europeanized African American literature. Why would he? That’s
not his topic.

(E), too, goes beyond the scope of the passage. The author has no interest in describing who
benefited more from exposure to the other group’s literature, Blacks or Whites.

• “Beyond the scope” choices, which are very common in LSAT Reading
Comprehension, may be tempting to you if you read them too quickly and
uncritically, or if you try to answer questions on a hunch rather than on an informed
re-reading of the relevant portion of text.

• Read carefully for time clues. If you note the 1862/1916 comparison in light of lines
17-19, (B) has to be correct; it’s the only choice that deals directly with the time
comparison.

© K A PL A N 5
LSAT PREP _________________________________________________________________ LSAT Test II Explained: Section I

5. (A)
One of the author’s basic points, revealed especially in lines 6-10 and 29-32, is that Black
writers adopted European literary canons in order to promote Black integration into White
society. Put another way, Black writers adopted these canons for reasons that were social
rather than artistic.

(B) and (E) Au contraire. According to the author, until Hughes came along to upset
tradition, the requirement that Black writers adopt White literary canons was both
important (B) and strong (E).

(C) Au contraire aussi. According to the author, Hughes’s success stemmed from his
unwillingness to follow this particular tradition.

(D) Unwarranted distinction. The author never mentions whether he considers the
adoption of White literary canons to have been appropriate in some cases and
inappropriate in others.

• Watch out for choices that clearly go against the basic thrust of the passage—three of
the choices in this case.

6. (D)
In lines 35-37, the author states outright that “Hughes’s expression of the vibrant folk
culture of Black people” rendered his work “a landmark.” This idea is reinforced in the
passage’s last sentence, too.

(A) is too broad, way beyond the scope of the passage, which deals only with Hughes’s
work in relation to earlier Black writing. For all we know, the author wouldn’t consider his
work novel in comparison to the work of later Black authors.

(B) There’s no reason to elevate this detail to center stage. Although “understatement” is
mentioned as an aspect of Hughes’s poetry (line 41), Hughes employed the Black folk
tradition of “hyperbole” (overstatement) as well.

(C) What musical forms did Hughes adapt to language? (C) really goes bonkers in
dragging ¶1’s musical references into the discussion of Hughes's poetic strengths.

(E) The passage never says that Hughes’s poetry had (or has) universal appeal. Also this
choice, like (B), elevates a minor detail (“street-talk rhyming”) to a major position.

• Beware of answer choices in global questions that grab onto minor details.

6 © K A PL A N
LSAT PREP _________________________________________________________________ LSAT Test II Explained: Section I

Passage 2—The U.S. Railroad


(Q. 7-13)
Topic and Scope: American railroads; specifically, the validity of Stilgoe’s argument
about how 19th century Americans viewed the railroad.

Purpose and Main Idea: The author’s purpose is to debunk Stilgoe’s “unconvincing”
argument about American attitudes toward the railroad; the author’s main idea is that,
contrary to what Stilgoe asserts, Americans were more enthusiastic than hostile toward the
railroad in the 1830s, but that some hostility continued in the 1880s and beyond.

Paragraph Structure: According to ¶1, Stilgoe argues that Americans viewed the railroad
differently in the 1830s than they did from the 1880s onward: In the 1830s, they distrusted
the railroad; from the 1880s forward, they were enthusiastic. According to the author, this
argument just doesn’t hold up under scrutiny. Everything else in ¶1 is simply background
information.

¶s 2 and 3 explain the author’s position. In ¶2, the author argues that Stilgoe draws an
unwarranted generalization about American distrust of the railroad in the 1830s. In fact,
she claims, only a small minority of intellectuals were unhappy with the railroad; the
general public viewed it as “an unqualified improvement.” In ¶3, the author claims that
Stilgoe is equally wrong to assert that unease with the railroad disappeared from the 1880s
onwards; she claims that, just as in the 1830s, a small minority of intellectuals remained
apprehensive about the railroad, even as the public at large applauded it.

The Big Picture:

• On Test Day, a passage like this one may be a good place to begin work on the
Reading Comprehension section. Topic, scope, and purpose are all plainly evident
by the end of the first ¶.

• On Test Day, you’re likely to see a “book review” passage similar to this one. Book
review passages usually have a classic set up in which the author either defends or
attacks someone else’s argument. What are they testing here? Read on:

• Make sure that you’re aware of the differences between the author’s opinions and
the opinions of others cited in the passage. The questions will almost certainly test to
see whether you can differentiate between authorial and non-authorial views.

• Be alert to time clues. The bulk of the passage becomes more manageable when you
see that the author is interested in comparing attitudes “both at [the railroad’s]
inception in the 1830s and. . . between 1880 and 1930.”

© K A PL A N 7
LSAT PREP _________________________________________________________________ LSAT Test II Explained: Section I

The Questions:

7. (E)
¶3 cites several pro-railroad post-1880s writers (Norris, O’Neill, and Adams; see Q. 11), but
has nothing to say about the particular arguments presented by these authors. Indeed, the
implication of lines 51-53 is that Stilgoe didn’t cite these writers.

(A) Lines 9-12 say that the United States rail system reached the “zenith of its popularity”
during the period that lasted from 1880 to 1930.

(B) Lines 2-5 reveal that the railroad had a “far-reaching impact” on the Industrial
Revolution in the United States.

(C) According to ¶2, Thoreau and Hawthorne are two writers who were uneasy about the
railroad during the 1830s.

(D) According to lines 51-53, Stilgoe could have strengthened his argument by citing the
works of Frank Norris, Eugene O’Neill, and Henry Adams.

• “All of the following EXCEPT” questions are best tackled by anticipating what must
be true of the exception. Here, we should realize that since four of the questions are
answered, the “odd man out” is almost certainly outside the passage’s scope. . . and if
you look for something that’s beyond the author’s interest, perhaps you locate (E)
faster than you would otherwise.

• In any event, do not anticipate that a passage’s first question must be low-difficulty.
In fact, the LSAT writers are notorious for mixing the difficulty levels of Reading
Comp. questions unpredictably. If, on a quick skim, a late question seems easy, grab
it; come back to tougher ones later on.

8. (B)
Stilgoe’s phrase “romantic-era distrust” is shorthand for his argument that 1830s Americans
in general were uneasy about the railroad. Remember, it’s a “distrust . . . that he [Stilgoe]
claims was present during the 1830s” (lines 13-14). Later on we learn the truth of the
matter—that the era was in fact pro-railroad, and only a few cranky intellectuals begged to
differ.

(A) and (D) No, it’s our author who believes that the distrust was a minority view held by a
few eggheads. Stilgoe ascribes distrust to the country at large, as (B) states.

(C) and (E) Au contraire, Stilgoe claims that there was no unease about the railroad from the
1880s onwards; his whole point is that the 1830s’ distrust turned to approbation later. But
according to our author, he’s as wrong about the later period as he is about the earlier.

8 © K A PL A N
LSAT PREP _________________________________________________________________ LSAT Test II Explained: Section I

• Here’s a prime example of a question in which you need to understand the


differences between an authorial and a non-authorial view. If you didn’t grasp the
differences between the author’s and Stilgoe’s positions, this question was probably
very confusing to you. Take great care, when you encounter a “book review”
passage, to sort out the broad points of view before tackling the questions.

9. (B)
Lines 46-51 say that Stilgoe’s own research revealed that post-1880s intellectuals like Henry
James, Sinclair Lewis, and F. Scott Fitzgerald displayed the same sort of “contrariety and
ambivalence” toward the railroad as was shown by such 1830s counterparts as Thoreau
and, inferably, Hawthorne.

(A) How could James, Lewis, and Fitzgerald have been influenced by the writings of
Norris, O’Neill, and Adams when the latter trio exhibited much warmer feelings for the
railroad?

(C) Most works of popular culture in the post-1880s era—contrary to the writings of
intellectuals like James, Lewis, and Fitzgerald—were happy about the railroad, says the
first half of ¶3.

(D) The author never suggests that James’s, Lewis’s, and Fitzgerald’s opinions about the
railroad were influenced by the opinions of earlier critics. Moreover, the word “reaction”
suggests that their opinions differed from those of the earlier critics, but that’s not the case.

(E) Once again, according to our author, most people in the 1830s were enthusiastic about
the railroad.

• In passages in which the opinions of a lot of different individuals or groups are


mentioned, be sure that you can differentiate among these opinions. You’ll likely be
asked to do so.

10. (E)
Lines 36-39 make it clear that the phrase “works of popular culture” refers to the post-1880s
works of “hitherto unknown” journalists, novelists, and illustrators that Stilgoe discovered
during his research. (E) paraphrases all this pretty closely.

(A) Post-1880s “works of popular culture” were supportive of the railroad, according to
both Stilgoe and the author.

(B) How could these “works of popular culture,” which were supportive of the railroad,
have been influenced by the works of writers who were uneasy about it?

(C) Adams, Lewis, and O’Neill are identified as intellectuals, not producers of “works of
popular culture.” Note that this choice curiously grabs two names from the enthusiastic
trio and one from the ambivalent trio. Isn’t that peculiar?

© K A PL A N 9
LSAT PREP _________________________________________________________________ LSAT Test II Explained: Section I

(D) looks great up until “1830s.” The phrase in question refers to journalists, novelists, and
illustrators working in the post-1880s era. Wrong era, wrong ¶.

• Wrong choices are often wrong for subtle reasons—like (D) here. Read a choice fully
and carefully before endorsing it.

11. (C)
Lines 51-53 indicate that Norris, O’Neill, and Adams were much more supportive of the
railroad than the earlier-mentioned James, Lewis, and Fitzgerald: that’s why Stilgoe’s case
(for general pro-railroad enthusiasm in the 1880s) would have been strengthened by citing
the opinions of Norris et al.

(A) is beyond the scope of the passage, which concerns attitudes toward the railroad, not
the popular appeal (or lack of same) of post-1880s intellectuals.

(B) The passage identifies Norris, O’Neill, and Adams as post-1880s intellectuals, so how
could their ideas have been disseminated in “the early 1800s”?

(D) is also beyond the scope of the passage. We’re never told exactly what kind of writing
these men did.

(E) is half-right, half-wrong. These three writers did feel somewhat differently about the
railroad from Thoreau, but the passage never says that their work was influenced by
Thoreau’s in any way.

• Earlier notes have discussed the presence of time clues in a passage. Such clues are a
sure signal that one or more questions will test whether you’ve got the passage’s
chronology down. (Questions 4, 8, and 10 are all of the same ilk.) Watch out for trap
choices—like (B) here—that confuse dates.

12. (D)
In ¶3, the author criticizes Stilgoe for relying on “works of popular culture” in drawing
conclusions about post-1880s societal attitudes toward the railroad. Hence, Stilgoe would
certainly acknowledge the validity of using such works to infer public attitudes.

(A) Not at all. According to Stilgoe, the “romantic era” refers to a specific era of American
history, the era that includes the 1830s.

(B) If anything, Stilgoe implies that the beliefs of intellectuals don’t necessarily foreshadow
the beliefs of popular culture. After all, he points out that the popular culture of the post-
1880s era was for the railroad even though some 1830s intellectuals were uneasy about it.

(C) This passage isn’t about “unpopular” writers, so we have no way of knowing what
Stilgoe thinks about the accuracy of such writers’ observations. (This choice is probably
trying to play on the reference to “hitherto unknown” railroad devotees (line 38), but even
if you decide that unknown = unpopular, Stilgoe uses those writers to tell a lot about their
era.)

10 © K A PL A N
LSAT PREP _________________________________________________________________ LSAT Test II Explained: Section I

(E) is wholly beyond the scope of the passage, which concerns attitudes toward the
railroad, not the Industrial Revolution in general. Tempting to those who have assimilated
only what they read in the first 10 lines or so.

• The correct answer to many LSAT Reading Comprehension questions is based on


information that’s in the middle, or near the end of, the passage, so you must never
assume that what comes after the first 1/3 isn’t important. (The point made in class is
that the first 1/3 lays the important groundwork for what will follow; that’s why we
give it great, but not exclusive, emphasis. See Q. 13, below, in this regard.) Read the
entire passage purposefully.

13. (A)
By the end of ¶1 we are wholly in possession of the author’s purpose: to take issue with
Stilgoe’s view of public attitudes toward the railroad from the 1830s to 1930. ¶s 2 and 3 go
on to explain just why the author finds Stilgoe’s view “unconvincing.”

(B) The scope of this passage covers public attitudes toward the railroad. Any literary
treatment cited by the passage (or by Stilgoe) is there to reflect public attitudes and not, as
(B) implies, for its own sake.

(C) is marginally o.k. up through the phrase “regarding the”—marginal only, since Stilgoe
is the only bona fide cultural historian cited here. (Thoreau et al are cited as social critics.) Be
that as it may, if (C) went on to read “regarding the U.S. public’s view of the railroad” it
might have something, but as is, (C) careens wildly outside the scope.

(D) This passage gives equal time to an era—the 1830s—when the national rail system was
not yet complete. Public support for the railroad predates the completion of the system by
a good 100 years.

(E) If there’s any ambiguity in the term “romantic-era distrust,” it’s eliminated by line 21.
(E) certainly cannot be the purpose of the passage as a whole.

• The correct answer to Global questions must be broad enough to encompass the
contents of the entire passage, yet narrow enough to refer specifically to those
contents.

© K A PL A N 11
LSAT PREP _________________________________________________________________ LSAT Test II Explained: Section I

Passage 3—Water Bug Adaptation


(Q. 14-21)
Topic and Scope: Developmental responses; specifically, water bugs as an illustration of
how organisms can adapt permanently and irreversibly to changing environmental
conditions.

Purpose and Main Idea: The author’s purpose is to describe three types of adaptations
that organisms may have in response to changing environmental conditions, and to discuss
one of those responses in depth through an extended example. Since this is a descriptive
(rather than argumentative) passage, the author doesn’t set forth any particular “main
idea.”

Paragraph Structure: ¶1 names three types of adaptive responses to changing


environmental conditions—regulatory, acclimatory, and developmental—but only
discusses the first two. ¶s 2 and 3 pick up the author’s real concern, by discussing in great
detail how developmental responses work in the case of water bugs. Specifically, ¶2
describes the two different water bug generations and the differences in their wing spans,
and ¶3 explains how environment accounts for those differences.

The Big Picture:

• This science passage is about as complex as they get on the LSAT. In the unlikely
event that you get a passage as intricate as this one, it’s probably best to leave it for
last, unless you’re confident that you can get through it quickly and accurately.

• Despite their complexity, passages like this one generally boil down to a few simple
relationships, and those are all you really need to grasp in order to cope with the
lion’s share of the questions. Don’t be overly concerned about assimilating all of the
jargon and scientific facts. If you need to know that stuff to answer a question or
two, you can always look it up.

• One useful way to get scientific relationships straight in your mind is to make a
mental picture of them. Here, you might have tried to picture the relationship
between water-bug wing size and season—for example, mentally seeing the need for
some long-winged bugs to fly away to water when a summer pond dries up.

• Beware of authorial curves! Through line 16 it appears as if regulatory and


acclimatory responses—call them “Moe” and “Larry” if you will—will have
important roles to play in the passage. The ‘heads up’ reader recognizes that from
line 16 to 66, this is all about Curly.

12 © K A PL A N
LSAT PREP _________________________________________________________________ LSAT Test II Explained: Section I

The Questions:

14. (D)
Exactly what we cited in “Paragraph Structure,” above. A choice ripe for pre-phrasing.

(A) is beyond the scope, which is a morphological rather than a functional adjustment.
(You don’t have to know these terms; we’re talking about how the author uses them in ¶1.)
This isn’t about how bugs use their wings, but what type they develop and why.

(B) The author states that there are three basis responses but never proves it. Proof requires
evidence, and we don’t get any. The evidence we do get is about how and why the
environment influences water bugs’ wings.

(C) focuses on details in ¶s 2 and 3. The micropterous vs. macropterous distinction is just
one part of the author’s larger discussion.

(E) The author never contrasts acclimatory and developmental responses. In fact, other
than simply defining an acclimatory response in ¶1, she has nothing more to say about it.

• LSAT authors have one of two basic purposes in mind—to argue something or to
describe something. When answering global questions about the author’s purpose
make sure that the main verb is truly consistent with the author’s purpose.

• Whenever you can, pre-phrase Reading Comp. right answers before looking at the
choices. This is especially important in Global questions, because you need to know
that your summation of a passage’s gist is accurate. You gain that knowledge by
comparing your summation to an answer choice such as (D) here.

15. (D)
Lines 31-42 say that the summer generation of water bugs is dimorphic—i.e., some bugs
develop long wings for flight (macropterous), while others develop stunted wings that
can’t support flight (micropterous). Why is this generation dimorphic? Well, picture it:
Most of the bugs stay near ponds and have no need to fly away, ever. But small ponds
sometimes dry up in summer, and so some bugs must be able to fly in order to find new
ponds to live in. In other words, dimorphism is a survival mechanism that ensures at least
some members of the summer generation will survive to procreate.

(A) and (C) The drying-up of a pond would mean that some bugs (the ones that couldn’t
fly away toward water) wouldn’t procreate at all, being dead; but it would have no
unusual effect on the wings of the next generation (C). Those wings would be as
macropterous as the overwintering brood’s wings always are. (Remember, heat leads to
normal wings; cold leads to stunted, flightless wings.) And of course, the number of
responses wouldn’t change at all (A)—just the number of living water bugs.

(B) The last we hear of the acclimatory response is line 14.

© K A PL A N 13
LSAT PREP _________________________________________________________________ LSAT Test II Explained: Section I

(E) In lines 45-46, the author explicitly mentions that the dimorphism of the summer
generation of water bugs has nothing to do with genetic processes.

• In a passage as complex as this one, you must go back and reread the relevant
portion of text before answering questions. Don’t try to cut corners here!

• Of course, you always have the option of skipping a toughie in favor of easier ones.
Here, for instance, Qs. 18, 20, and 21 are arguably considerably easier than Q. 15 and
the rest. It’s a pity to skimp on the three easier ones—or worse, to skip them
altogether—simply because you got hung up earlier on.

16. (A)
Lines 49-50 say that warm temperatures lead to water bugs with normal wings—i.e.,
macropterous bugs. Hence, it follows that a warmer than usual winter would produce a
higher than normal proportion of macropterous—normal-winged—bugs.

(B) Au contraire. Higher temperatures, we’ve just noted, would increase the proportion of
macropterous (not micropterous) individuals.

(C) The thrust of ¶s 2 and 3 is that temperature change has a palpable effect on water-bug
development. That’s what it’s all about!

(D) and (E) are beyond the scope. There’s no information to suggest that warmer than usual
winter temperatures would affect either water-bug behavior (D) or egg formation (E).

• This question highlights the importance of getting scientific relationships straight


in your mind. If you had the relationship between temperature and wing formation
down, this question should have led to a quick and easy point.

17. (A)
Lines 55-63 reveal that eggs produced in autumn are exposed to the cold “and thus
produce micropterous adults in the summer generation,” while those eggs produced in
the spring are not exposed to cold “and thus yield individuals with normal wings” (i.e.,
macropterous individuals). And there you have it: the dimorphic wing structure
displayed by the summer generation.

(B) The eggs are exactly the same; it’s the different temperatures to which they are exposed
that leads to the morphological difference in adult water bugs.

(C) No, seasonal changes lead to a developmental morphological response that affects wing
structure. See how they keep trying to drag in “acclimatory”?

(D) No, the reason that the summer generation includes individuals with both wing
structures is that sometimes summer generation water bugs must fly to reach new habitats.

14 © K A PL A N
LSAT PREP _________________________________________________________________ LSAT Test II Explained: Section I

(E) The habitat of the overwintering generation has nothing to do with the summer
generation’s dimorphic wing structure. Again, it’s the different temperatures to which the
overwintering generation’s eggs are exposed that causes this morphological reaction.

• This question, like #16, plays on the temperature and wing structure relationship.
Notice that none of the wrong choices addresses this relationship.

18. (B)
Lines 6-10 indicate that regulatory responses involve changes in the rate at which an
organism’s systems function. An increase in pulse rate would qualify as a regulatory
response. (You might even have pre-phrased “heartbeat” as the right answer after reading
lines 6-10.)

(A), (C), (D), (E) All of these examples are acclimatory responses—reversible changes that
alter the organism’s physiology.

• Know what’s going on in each ¶ in order to localize the answers to individual


questions. Here, for instance, those who recorded the location of various details as
they read knew that the answer must come out of ¶1 only: It’s the only ¶ that deals
with regulatory responses.

19. (E)
Water bugs hatched during the summer are macropterous, since the eggs from which they
emerge are never exposed to cold temperatures. In contrast, the early-spring generation
contains some eggs that are exposed to cold temperatures; therefore, some of this
generation are micropterous.

(A) No, water bugs hatched during the summer are all macropterous.

(B) This generation of water bugs lays its eggs at two distinct times: early autumn and
early spring. Only the autumn eggs are exposed to cold; the spring eggs are not.

(C) Since this generation of water bugs is macropterous (i.e. has normal wings), its
members are capable of flight from one pond to another.

(D) Yes, there is a difference in wing form between this generation and that of early spring,
but this difference is caused by temperature changes in the environment. Lest we forget,
genetics are explicitly excluded from having an impact on wings (lines 43-46).

• When dealing with passages that contain a lot of scientific jargon, it’s usually
helpful to jot down some very quick shorthand in the margins. For example,
alongside this passage, you might have jotted down that macropterous=flighted
wings—eggs warm and micropterous=flightless wings= eggs cold.

© K A PL A N 15
LSAT PREP _________________________________________________________________ LSAT Test II Explained: Section I

20. (C)
In lines 58-61, the author notes that water-bug eggs exposed to cold winter temperatures
produce micropterous individuals. But when these same eggs are taken out of the cold
and put into a warm laboratory, they produce macropterous individuals. In other words,
the purpose of mentioning the experiment is to show that temperature clearly has an effect
on the development of water-bug eggs.

(A) refers to the wrong part of the passage. The function of the summer generation’s
dimorphism is explained in ¶2.

(B) is outside the scope of the passage, which concerns one and only one developmental
response—wing development.

(D) No, the laboratory experiment concentrates on how the same generation’s eggs may
develop differently in response to different temperatures; it doesn’t address differences
between water-bug generations.

(E) This passage discusses a morphological, not a functional, adjustment.

• When a question asks why the author included a particular detail, the answer is
likely to be found in the lines around that detail. In most cases, choices that are
based on information from a different part of the passage—like (A) here—will be
wrong.

21. (A)
¶1 introduces three biological phenomena—regulatory, acclimatory, and developmental
adaptive responses—and discusses examples of the first two. ¶s 2 and 3 then go on to give a
very detailed example of the third type of response.

(B) Only one biological phenomenon—developmental adaptive response—is discussed in


any detail.

(C) The passage never presents any hypothesis linking regulatory, acclimatory, and
developmental responses. It simply presents them as three possible responses to changing
environmental circumstances.

(D) The passage never claims that the three responses are “complementary.” Furthermore,
all have the same cause: changing environmental conditions.

(E) What “new” way of describing biological phenomena? And what “applications” are
presented?

• Sometimes the very last question on a passage can be its easiest.

16 © K A PL A N
LSAT PREP _________________________________________________________________ LSAT Test II Explained: Section I

Passage 4—War Powers Resolution


(Q. 22-28)
Topic and Scope: The War Powers Resolution; specifically, how this resolution has
enhanced Congress’s role in decisions to send American troops into undeclared wars.

Purpose and Main Idea: The author’s purpose is to describe the War Powers Resolution,
its political background, and its consequences. His main idea is that this resolution has
strengthened Congress’s role in decisions that involve sending American troops into
undeclared wars.

Paragraph Structure: ¶1 tells us that the Constitution doesn’t explicitly define either the
President’s or the Congress’s powers to deploy troops to undeclared wars. Nevertheless,
the author is quite certain that the “spirit” of the Constitution mandates some role for
Congress in these decisions.

¶2 provides yet more background, telling us that, until the War Powers Resolution was
passed, Presidents routinely deployed American troops to undeclared wars without the
consent of Congress.

¶3 describes the provisions of the War Powers Resolution itself, explaining how it limits
the President’s authority to involve America in undeclared wars.

The Big Picture:

• Keeping track of details by noting paragraph topics is a helpful strategy that’ll save
you valuable time when you need to relocate details to answer questions. As you
read through this passage, you should have noted that the first two paragraphs deal
mainly with background information: ¶1 with legal background info and ¶2 with
historical background info. You should also have noted that ¶3 deals exclusively
with the War Powers Resolution itself.

The Questions:

22. (B)
At the end of ¶1, the author explicitly states that the War Powers Resolution is an attempt
by Congress to reclaim its rightful role in decisions to involve America in undeclared
wars. In ¶s 2 and 3, the author demonstrates that Congress lost this role, but has now
regained it. You may have found “usurped” a mite extreme, but given the author’s firm
conviction that the Constitution demands Congressional involvement in troop decisions,
”usurpation” is pretty close to the behavior we hear about in ¶2.

(A) The Vietnam War’s role in leading to the War Powers Resolution is merely a ¶2 detail.

© K A PL A N 17
LSAT PREP _________________________________________________________________ LSAT Test II Explained: Section I

(C) and (D) are beyond the scope of the passage, which is restricted to a discussion of
Congress’s role in decisions to send American troops into undeclared wars. This passage
doesn’t delve into the broader Presidential-Congressional struggle for control of the
military (C), let alone the even broader issue of the Constitutional balance of power
between the executive and legislative branches of government (D).

(E) Lines 58-60 explicitly state that the War Powers Resolution isn’t intended to alter the
Constitution.

• When answering Global questions, steer clear of choices that violate the passage’s
scope.

• Don’t nickel-and-dime the answer choices. The 4 wrong choices have been set up
with manifest problems, while the 1 right answer has been set up to be manifestly
strong overall. You may not be completely crazy about its wording, but beware of
rejecting the credited choice on nit-picky grounds.

23. (E)
In lines 20-22, the author argues that the Constitution’s spirit, if not its precise wording,
mandates that Congress should take part in the decision to send soldiers in an undeclared
war. The President’s role, of course, is mandated by his Constitutional status as
commander in chief. In other words, the author thinks that the Constitution requires a joint
decision by the President and the Congress.

(A) The author says that the Constitution mandates a joint decision. That’s very different
from saying that he thinks the Constitution assumes agreement between the President and
the Congress. Besides, if they were sure to agree, what would be the point of mandating
joint consultation?

(B) Au contraire. According to the author, the Constitution isn’t explicit about the division
of authority in decisions to deploy troops in undeclared wars.

(C) and (D) Since the author argues that the Constitution doesn’t clearly lay out the
President’s and the Congress’s powers with respect to decisions to involve America in
undeclared wars, he can’t possibly conclude that either Congress (C) or the President (D)
has a greater role in these decisions.

• If you answered Question 22 confidently, then here in #23 choice (E) should have
jumped out at you. The answer to one question may often unlock the answer to
another question in the set. If you get stuck on a question, see whether your work on
the other questions can get you closer to the correct answer.

18 © K A PL A N
LSAT PREP _________________________________________________________________ LSAT Test II Explained: Section I

24. (B)
The only point the passage makes about defense treaties is that the President is obligated to
uphold them. It never suggests that (B) Congress enacted the War Powers Resolution
because Presidents failed to execute this duty.

(A), (C), (D), (E) Lines 36-47 cite the issues raised in these choices as contributing factors in
the Congress’s decision to enact the War Powers Resolution.

• In all/EXCEPT questions, work confidently. Four of the choices share common


traits—they’re all true, or they all come from the same part of the passage. Look
boldly for the “odd man out.”

25. (B)
Lines 8-10 indicate that Congress has the right to declare war, and by giving the President
that green light to go to war Congress is ipso facto involved in the decision to deploy
American troops. In other words, because Congress participates in the initial decision to
send American troops to war, the War Powers Resolution is irrelevant in cases in which
war is formally declared. (And notice that the scope of this investigation—namely, troop
involvement when war is not declared—is made clear as early as lines 4-5, where we find,
word for word, the very same phrase that the stem cites.)

(A) What other laws? The passage doesn’t mention any. Anyhow, it’s the Constitution that
has set out Presidential requirements when war is declared.

The remaining choices are all beyond the scope of the passage. The text contains no
discussion of public opinion in declared wars (C); Congress’s attitudes toward the
President’s behavior in declared wars (D); or Presidential reporting and consulting
procedures in declared wars (E). And why should the text discuss any of those things?—
none of them relate to our author’s investigation as defined in the first few lines.

• Choices that fall outside the scope of the passage are very common in Reading
Comprehension generally, and especially in inference questions. For that reason:

• Be very careful to note early on—and keep in mind throughout—the author’s scope.

© K A PL A N 19
LSAT PREP _________________________________________________________________ LSAT Test II Explained: Section I

26. (C)
Lines 20-25 echo the sentiment expressed in this choice.

(A) If anything, the author would argue that the War Powers Resolution is consistent with
the roles of the President and the Congress, at least given how the author perceives the
spirit of the Constitution.

(B) No, the author maintains that the Congress has a legitimate role in making decisions
about troop deployment in undeclared wars.

(D) Au contraire. The author feels that the War Powers Resolution fills an important legal
gap concerning America’s response to undeclared wars.

(E) Au contraire aussi. The author argues that the War Powers Resolution undermines “the
role historically assumed by Presidents” (that of usurper), by giving Congress a voice in
decisions that were previously the President’s prerogative.

• Sometimes the correct answer to an inference question is nothing more than a mere
restatement of an idea expressed in the passage. You don’t have to work equally hard
for every LSAT point!

27. (A)
In ¶2, the author identifies the Cambodia invasion as an instance when a President
deployed American troops in an undeclared war without approval from the Congress.
Any decision to deploy troops in an undeclared war that’s not approved by Congress, he
claims in ¶1, violates the “intent and spirit of the Constitution.”

(B) is a “half-right, half-wrong” choice. True, the Cambodia invasion did contribute to
support for the War Powers Resolution; but this resolution curbed, rather than expanded,
the President’s power.

(C) The author never says or suggests that the Cambodia invasion “was necessitated by a
defense treaty,” nor does he describe any such cause-and-effect as (C) suggests.

(D) The War Powers Resolution, ¶3 makes clear, wasn’t meant to challenge the President’s
authority under the Constitution.

(E) If anything, the author suggests that the Cambodia invasion followed a historical
pattern established soon after America’s birth.

• Choice (B) reveals the importance of reading your chosen answers fully and
carefully. It looks pretty convincing to those who read just the first half. Always read
your selection carefully and thoroughly before accepting it. (If you can give the other
choices a similar stern evaluation within timing limits, so much the better.)

20 © K A PL A N
LSAT PREP _________________________________________________________________ LSAT Test II Explained: Section I

28. (D)
The answer must come from ¶3 only. There we see that lines 51-53 state that the President
must “report to Congress within 48 hours after . . . forces have actually been deployed”
under the provisions of the War Powers Resolution.

(A) Same old scope ground. The War Powers Resolution is specifically for situations in
which a formal declaration of war isn’t contemplated.

(B) Lines 49-51 say that the President must consult with the Congress “in every possible
instance” before deploying troops. That’s not quite the same as saying that the President
must consult in every instance. The resolution recognizes that there may be circumstances
in which the President must deploy troops before talking to the Congress.

(C) and (E) The resolution allows the President to deploy troops for up to 60 days without
the Congress’s approval (C). And it leaves open the prospect of even longer unsanctioned
deployments in certain instances (E).

• If you didn’t go back to carefully reread (or at least skim) ¶3, all of the choices may
have seemed plausible. When in doubt, go back and reread. Doing so usually
doesn’t take that much time, and is likely to result in a better score.

© K A PL A N 21
SECTION II:
LOGICAL REASONING

22 © K A PL A N
LSAT PREP ________________________________________________________________ LSAT Test II Explained: Section II

1. (D)
The author in the passage reasons by analogy. The question posed in the second sentence is
obviously meant to be absurd; by extension, the author means to imply that the claim in the
first sentence is absurd as well. Once you recognize this argumentative structure, you can
simply scan the choices looking for an analogy used in the same way. Choice (D) is the
only choice that contains a similar analogy; the author compares the idea that economists
can control inflation to the idea that meteorologists can control the weather. As in the
original, the second idea is ridiculous and is used to imply that the first idea must be
incorrect as well.

(A) Unlike the stimulus, the answer to the question in the second sentence here is
supposed to be “yes.” In other words, in this short argument the second sentence is
supposed to lend support to the first claim.

(B) The question in the second sentence doesn’t set up an analogous situation; rather it
directly attacks the initial idea.

(C) No; here the question in the second sentence serves to explore the claim made in the
first sentence. If we knew whether the sports were managed by the same type of people,
we’d be in a better position to evaluate the assertion that they’re both fixed. The big
problem here, as in (B), is that there’s no analogy being made.

(E) Here, the question is supposed to suggest a counterexample to the initial assertion, but
once again, it argues directly from the terms of the first sentence, whereas we’re looking for
a seemingly absurd analogous situation in the question posed in the second sentence.

• You may have felt that the analogy in the stimulus wasn’t a very good one; the
second idea is obviously much wackier than the first. Hold on to that! You’ll notice
the analogy in correct choice (D) is also shaky. A good parallel should mirror all the
peculiarities of the stimulus argument.

• In Parallel Reasoning questions, always attempt to abstract from the original; that is,
try to paraphrase the argument in general terms. If you were able to recognize the
original’s second sentence as an absurd analogy, you should have been able to spot
the correct answer, whether or not you even realized what function this analogy was
supposed to serve (as stated above, the function is to imply that the belief in the first
sentence is silly).

2. (E)
Bill’s statement is clearly meant to refute Ann’s remark; therefore, his statement must
contradict the statement he thinks Ann made. He says that there are some Tri-Cities
students who are campers at Camp Lakemont. What would that contradict? How about the
claim made by (E), that only campers at Camp Winnehatchee are Tri-Cities students? If Bill
thought that Ann was making this claim, then he could indeed refute it by pointing out
that there are also campers at Camp Lakemont who are Tri-Cities students.

© K A PL A N 23
LSAT PREP ________________________________________________________________ LSAT Test II Explained: Section II

(A) Bill doesn’t claim that the majority of campers at Camp Lakemont are from Tri-Cities
High School, so his statement isn’t intended to rebut (A).

(B) Bill never discusses what proportion of Tri-Cities students camp at Camp
Winnehatchee as opposed to Camp Lakemont (or any other camp), so he can’t be trying to
refute (B).

(C) The idea of “withdrawing” from camp never comes up; Bill merely says that some Tri-
Cities students go to Camp Lakemont, which doesn’t imply that none of them has ever
withdrawn.

(D)’s all wet; how would Bill’s claim that some Tri-Cities students go to Camp Lakemont
weaken the claim that all Tri-Cities students go to summer camp?

• Here’s a trick for this type of “crossed-wires” question. We’re looking for Bill’s
misinterpretation of Ann’s statement. Ann never even mentions “Camp Lakemont,”
so Bill can’t think that she was saying anything about Camp Lakemont. That
immediately eliminates (A) and (C). The misinterpretation must be plausible: it must
deal with the same ideas that the first speaker deals with.

3. (E)
What’s a police force to do? That’s the topic here; the police force of this nameless city
seems to be all over the map. First the city decides that the emphasis will be switched from
writing speeding tickets to writing parking tickets. Fine. But with no signs of a crackdown
on illegally parked cars, the police chief explains that the resources devoted to speeding
tickets instead has been diverted to the war on drugs. Even with this development,
however, the number of speeding tickets remains the same. The author adds all this up
thusly: the police chief’s claim that resources have been shifted to fighting drugs must be
false (a conclusion signaled nicely by the word “Therefore” in the last sentence). In order
for this conclusion to stand, the author must implicitly believe that if resources had really
shifted to fighting drugs, then the number of speeding tickets would have to drop. But is
that so? If it were possible for the police to continue writing as many speeding tickets as
ever while still diverting resources to fighting drugs (which is the denial of (E)), then the
argument would be defeated. If we deny (E), the argument falls apart, confirming (E) as the
necessary assumption we seek.

(A) and (B) are both irrelevant. Use the Kaplan Denial Test to help you out. Suppose that
some officers were unqualified to work on fighting drugs, or that the drug problem is just
as serious as the police chief claims. So what? Neither of these things damages the author’s
conclusion that the police chief’s claim is false, which shows that neither (A) nor (B) is
necessary to the argument.

(C) is out of the scope. The issue is the actual allocation of resources in the city, and not
how resources should be allocated. This is another version of the “can vs. should” scope
shift.

24 © K A PL A N
LSAT PREP ________________________________________________________________ LSAT Test II Explained: Section II

(D) is an au contraire choice—if the author assumed this, she’d have no business concluding
that the police chief is lying. For further proof from a different angle, let’s look at (D)’s
opposite. If it were impossible for the police to fight drugs and the illegal parking scourge
without reducing the number of speeding tickets, then the author’s argument would be
strengthened. Since the opposite of (D) would strengthen, not damage, the argument, (D)
can hardly be necessary to the argument.

• Use Keywords to help you identify the conclusion and structure of the argument. In
the last sentence, the Keyword “therefore” tells us the author’s conclusion will
follow. Also, the Keywords “yet” and “but” indicate a contrast that the author uses to
suggest a conflict.

• Use the Kaplan Denial Test to find necessary assumptions and eliminate wrong
answers. The negation of the correct answer in an Assumption question will defeat
the argument. On the other hand, if the negation of an answer choice fails to defeat
the argument (or even strengthens the argument, as in (D) here), then that choice
cannot be necessary to the argument.

4. (D)
Why would dried grass clippings be better for plant growth than fresh grass clippings? We
know that dried clippings decompose gradually, providing nutrients, but the stimulus
provides no information about the qualities of fresh grass clippings. Choice (D) solves the
mystery. If the rapid decomposition of fresh clipping generates heat that kills beneficial
bacteria, then dried clippings, which decompose gradually, would provide better results.

(A) and (E), if true, apply equally well to both dried and fresh grass clippings, and so they
cannot explain why one would be better than the other.

(B) This is all well and good, and supports the notion expressed in the stimulus that dried
grass clippings are good for beneficial soil bacteria and plant growth. However, what this
choice fails to do is explain why dried clippings are more effective than fresh clippings,
which, for all we know, could have the same qualities.

(C) has the same problem as answer choice (B)—continuing to heap praise on dried grass
clippings doesn’t tell us the one thing we want to know: why is this kind of clipping better
for plant growth than fresh grass clippings? Stating the benefits of dried grass clippings by
itself does not explain the mystery, because for all we know the fresh grass clippings have
the same benefits. We need an explicit difference between the two, which choice (C) doesn’t
provide. Moreover, (C) applies to some dried clippings, whereas the stimulus applies to
dried grass clippings in general.

© K A PL A N 25
LSAT PREP ________________________________________________________________ LSAT Test II Explained: Section II

• Watch out for choices in explanation questions that could be true but still wouldn’t
help to explain the result. Here, all the wrong choices could be true, but none of
them explains why dried clippings would be better than fresh ones.

• If the question stem asks you for a reason why one candidate is better than another,
then choices that apply equally well to both (here, (A) and (E)) must be wrong.

5. (C)
What’s wrong with this picture? The author predicts that a gasoline tax will raise fifty
billion dollars a year, based on current consumption rates. However, the author also
predicts that the tax will reduce demand for oil, and hence consumption rates, which will in
turn lead to other benefits. Well, our author can’t have it both ways—one of those
predictions has got to go. Assuming that consumption rates will remain the same is
incompatible with assuming that a tax will lower those same rates, and so (C) identifies the
error in the author’s reasoning.

(A) All the data cited in the stimulus is relevant: it all deals in some way with the
economic factors related to a gasoline tax. The argument is flawed, but in another way.

(B) We have no reason to believe that the author’s consumption figures are incorrect.

(D) If (D) were correct, then one of the causal relationships in the stimulus would have to
be backwards, meaning that one of the results cited by the author would have to really be a
cause, and vice-versa. The reasoning has a problem, but this isn’t it.

(E) An “appeal to conscience” entails trying to sway people by making them feel guilty or
by supplying some emotional justification (“a good patriotic citizen who cares anything
about his or her country would support this tax”) for a position rather than providing a
logical one. The author doesn’t argue in this way; she does appeal to reason (albeit badly).

• Always try to pre-phrase your answer and use it to scan the choices aggressively. If
you do, then you won’t be slowed down by complicated wrong answer choices such
as (D) here. It takes longer to understand exactly why (D) is wrong than it does to
simply recognize that it doesn’t match your pre-phrase.

6. (B)
This author pounds home the main point with each sentence. First: bald eagles have the
unique capacity to foster a sympathetic attitude toward other threatened species. Second:
without that sympathy, the needs of those other species will be unmet. Third: the needs of
other species can only be met by beginning with the conservation of the bald eagle. Clearly,
the author’s point is that it is first necessary to see to the conservation of the bald eagle if we
hope to be able to conserve any other threatened species; or, as (B) has it, the conservation
of the bald eagle is the necessary first step toward conserving other threatened species.

26 © K A PL A N
LSAT PREP ________________________________________________________________ LSAT Test II Explained: Section II

(A) The author doesn’t say conservation efforts should concentrate on bald eagles instead of
other species; she says they should concentrate on bald eagles because that’s the only way
other obscure endangered species can be saved.

(C) is a distortion of the author’s point; it makes only a vague reference to the bald eagle
(an important symbolic species), although never mentions it by name, and also ignores
entirely the conservation needs of the more obscure endangered species.

(D) is too vague—like (C), it doesn’t even mention the bald eagle, which was clearly the
focus of the argument.

(E) While (E) is technically correct in saying that the author gives special importance to the
bald eagle, this is not the author’s main point. It is, in fact evidence for her main point: that
because of the unique importance of the bald eagle, conservation of the bald eagle is a
necessary first step toward conserving other species.

• Use your Reading Comp. skills on the Logical Reasoning sections whenever
possible. You can think of this as a Main Point question for a mini RC passage. In
RC, you learn that the answer to a Main Point question must be broad enough to
cover the entire passage without leaving out the main elements of the passage. In this
case, choices that don’t contain the bald eagle (choices (C) and (D)) can be eliminated
immediately.

• The main point of the passage is not just some statement with which the author
would agree. Ask yourself: why is the author making an argument? What is he or she
trying to convince me of? That’s the main point.

• In Main Point questions, often there will be words or phrases in the correct choice
that paraphrase those used in the argument. Here, saying that the needs of X “can
only be met . . .” by Y is the same as saying that Y is “necessary” for X.

7. (A)
The author boldly announces her conclusion in the first sentence, which claims that
requiring official confirmation of scientific work serves no purpose at all. For evidence, the
author notes that poor scientific work will be exposed if other scientists conduct the same
experiments. That’s great, but how long will it take to expose shoddy work? If scientific
experiments can go unchallenged for years before they are replicated (choice (A)), then bad
scientific work could be in circulation causing damage for quite a while before it is
eventually retracted. This, contrary to the author’s contention, would give us a reason to
prefer official confirmation of scientific findings. (A) weakens the argument.

(B), if anything, could only strengthen the argument. If most scientific work is subject to
peer review, then “official confirmation” would seem less necessary, and the author’s
conclusion would seem more reasonable.

© K A PL A N 27
LSAT PREP ________________________________________________________________ LSAT Test II Explained: Section II

(C) is perfectly in line with the author’s ideas—the author believes that replication is the
way to expose bad science. However, this leaves open the time-frame problem identified in
(A): how long will it take to discover potentially harmful mistakes?

(D) offers an irrelevant distinction which has no bearing on the argument. It matters not in
the least which one of the stated causes of poor scientific work, careless reporting or fraud,
is more prevalent.

(E) is mostly irrelevant, but like (B), would, if anything, strengthen the argument. If most
scientists work as part of a team, then their work would probably be reviewed by other
scientists in that team, which would make official confirmation seem less necessary. A
convoluted argument, but in any case, (E) is certainly no weakener.

• Paraphrase and pre-phrase! The stimulus will give you enough information to
answer the question, but rarely in the form that is easiest to understand. The best
approach is to put the argument in your own words. Once you have a strong sense of
the argument, think of what the answer should look like, and then search
aggressively among the choices.

• Beware of irrelevant distinctions, such as the one that appears in (D)—this type of
fancy wrong answer choice shows up a lot on the test. To form this wrong answer
choice, the testmakers take two elements that appear in the stimulus and compare
them in a way that has no bearing whatsoever on the argument. In other words, they
pose a distinction that true, false, likely or unlikely is essentially irrelevant.

8. (E)
David attacks Alice’s recommendation for dropping quotas by pointing out that lots of
countries have such quotas, so the U.S. should have them too. Hopefully your response to
this irrelevant bit of nonsense was “Huh? So what? What’s your point, David?” How does
David address Alice’s argument that direct competition ultimately benefits the consumer,
the reason she suggests dropping quotas? He doesn’t—he sidesteps Alice’s point and
instead advances his own reason in favor of quotas. (E) correctly characterizes David’s
lame reply.

(A) David doesn’t accuse Alice of self-contradiction; he merely points out that she has
overlooked an important fact, a fact which he believes weakens her conclusion.

(B) Don’t be fooled; there is no personal attack here. Saying “you fail to realize” isn’t a
personal attack on Alice—it’s an attack on her argument.

(C) is a little trickier, but David isn’t pointing out that Alice assumes that other countries
don’t have quotas (and indeed her argument need make no such assumption). His point is
that she has overlooked the question of other countries’ quotas entirely.

28 © K A PL A N
LSAT PREP ________________________________________________________________ LSAT Test II Explained: Section II

(D) should have been the easiest choice to kill: clearly, from his first four words alone,
David is disagreeing with Alice.

• Read the argument and reply actively, as if LSAT dialogues represent a real-life
exchange of ideas. Trust your own reaction: did David’s reply seem like a good
answer to you? Did you wonder what happened to the “benefit of the consumer?”
Carry that reaction with you to the answer choices and look for a match.

• When testing a choice in a Method of Argument question, think about what the
argument would sound like if that particular choice were correct. For example, if (A)
were indeed David’s method, his argument would sound something like this: “Alice,
you say X, but then you also say Y, and these two things can’t both be true.” That’s
what an “accusation of contradiction” might sound like. A personal attack, (B),
would have to contain much stronger language than David actually employed— “It
figures you would say that, Alice, because you are ugly and stupid and have no idea
what is going on in the world . . .” —this would be closer to David’s argument if (B)
were meant to be correct. Analyzing the choices in this way will help you to identify
the one that does match the method of argument employed in the stimulus.

9. (B)
Rearranged, this stimulus becomes a simple logical chain, starting with the last sentence:
child-care services are socially necessary and are being publicly criticized, therefore the
government must respond. The government has only one response to criticism of socially
necessary services, and that response is to regulate the activity providing that service.
(Therefore, we can infer that the government will regulate the provision of child-care
services.) Every time the government regulates an activity, it inevitably makes that activity
more expensive. So finally, we can infer (B): the government will make the provision of
child-care services more expensive.

(A) The stimulus makes no reference to any difference in the quality of services regulated
by the government.

(C) We know the government will regulate child-care services, but we can’t be sure that
means “fostering advances” in child care. Moreover, it’s not clear to what “funding” (C)
refers; the stimulus never spoke of funding.

(D) goes beyond the scope. The stimulus said that the government inevitably responds to
criticism of socially necessary services by regulation. The stimulus never referred to
government response to criticism of policy in general.

(E) We know that if child-care services are regulated by the government, then their price
will increase. Logically, we can’t simply negate both terms and infer that if the services
aren’t regulated their price won’t increase. (See the second bullet point.)

© K A PL A N 29
LSAT PREP ________________________________________________________________ LSAT Test II Explained: Section II

• Never inject your own opinion into the argument. Your opinion about whether
government regulation improves child care matters not one whit. What matters is
only what appears in that 10-line stimulus, and neither “quality” nor “advances in
child care” is part of the story.

• The formal name for the incorrect inference in (E) is “Denying the Antecedent.” That
occurs when someone sees the statement “If A, then B” and infers from it “If NOT-A,
then NOT-B.” You don’t have to know the formal name, but you do have to recognize
this form of erroneous logic because it appears often on the LSAT. Be on the lookout
for this line of reasoning—it’s wrong, wrong, wrong. (Incidentally, the correct
inference, the contrapositive, is “If NOT-B, then NOT-A.” To form the contrapositive
of an if-then statement, you must reverse and negate the terms, not simply negate
them.)

• The testmakers don’t fool around with the meaning of words the way we’re prone to
in everyday life. When they use a word like “inevitably,” they don’t mean
“sometimes,” “most of the time,” “usually,” and so on. If something “inevitably”
follows from something else, that means “always”—the second will ALWAYS follow
from the first. That’s what “inevitably” means, and understanding that is
instrumental to getting this point. This is one way in which the testmakers test
whether or not you understand the strict definition and logical consequences of
what seem to be common everyday words.

10. (C)
Responding to the criticism of advertisers, the author argues that some of them are
influenced by morals as well as profit. To support her position, the author cites the case of
a publication that switched from a family orientation to a concentration on sex and
violence. In response, some advertisers withdrew their ads. The author concludes that they
must have withdrawn their ads because of their moral disapproval, but is that necessarily
so? Maybe they thought that advertising in the new “salacious” magazine would hurt them
financially. Maybe the advertisers’ decisions had nothing to do with moral considerations,
as the author posits; maybe, as always, it’s the Almighty Dollar that rules. This line of
reasoning would weaken the author’s argument, and the cynical among us may have been
thinking just this as we read through the stimulus.

However, we’re asked to strengthen, not weaken the argument, and one way to do that is to
shore up the potential weakness just uncovered, the possibility that financial
considerations outweighed moral ones in the advertisers’ decision. The author assumes that
this is not so, so essentially we’re attempting to strengthen the argument by bolstering the
author’s central assumption. (C) does just that: if the advertisers who left the publication
would have increased their sales had they stayed, then they couldn’t have left solely for
financial considerations; something else must have played a role. Thus, ruling out money
as the sole consideration in effect strengthens the author’s argument that the advertisers left
on moral grounds.

30 © K A PL A N
LSAT PREP ________________________________________________________________ LSAT Test II Explained: Section II

(A) If advertisers switched to other family newspapers, then we know that they prefer
family newspapers, but we don’t know why. They might think that advertising in family
newspapers will lead to more sales.

(B) is unrelated to the evidence, which concerns the advertisers that switched from the
publication, not to the publication.

(D) and (E) are out of the scope as well: the issue is the motivation of the advertisers, not the
motivation of the publishers, or the preferences of readers of family newspapers.

• Don’t rush through the stimulus—the first step in strengthening and weakening
arguments is to know what the argument is.

• When the stimulus indicates that something is “often” the case, expect the next
sentence to point out an exception. Conventional wisdom “often” takes a beating on
the LSAT. In this question, we are told that advertisers are often criticized for their
lack of morals, and so we should expect the author to hold a conflicting point of
view. Use the critical reading skills discussed in Unit 2 and throughout the Kaplan
course to help you anticipate where the author is about to go, not only in LR but in
Reading Comprehension as well.

• Often, the best way to strengthen an argument is to shore up a key assumption. By


the same token, the best way to weaken an argument is usually to undermine a key
assumption.

11. (B)
This is a fairly difficult question; many test-takers reported trouble with this one, and
much of that trouble began with the question stem. The first thing to realize is that there are
two requirements the correct answer must satisfy. The second requirement might be easier
to test than the first, so let’s start there. The correct choice must contradict (that’s what it
means to be “not consistent with”) the politician’s claim. Paraphrased, here’s what the
politician says: if deforestation is stopped, then the koala will NOT become extinct. Make
sure you understand that this is what the politician is saying; the key is the phrase “all that
is needed,” which denotes sufficiency: “stop deforestation, and the koala is saved,” is in
essence the politician’s claim.

(B) clearly contradicts this: in this choice, deforestation is stopped, and the koala does
become extinct. Now for the other requirement of the stem: is (B) consistent with the
biologist’s claim? Well, the biologist says only that continued deforestation at the same rate
would cause the approaching extinction of the koala; he never said that deforestation was
the only thing that could bring about that result. So (B) is consistent with the biologist’s
claim—while it doesn’t prove or even support this claim, it doesn’t contradict it either. (B)
therefore satisfies both requirements of the stem and is the correct answer.

© K A PL A N 31
LSAT PREP ________________________________________________________________ LSAT Test II Explained: Section II

(A) isn’t inconsistent with the politician’s statement, which only predicted the koala’s
survival if deforestation was stopped.

(C) “Reforestation” is beyond the scope; neither speaker says what will happen when
reforestation occurs. However, if you assume that the occurrence of “reforestation” means
that deforestation has indeed ceased (and that sounds plausible), then (C), rather than
opposing the politician’s argument, describes the very thing that the politician predicted.

(D) and (E) both speak of what happens when deforestation is “slowed,” i.e. continues but
at a slower rate. However, neither one contradicts the politician, who said only that stopping
deforestation would guarantee the koala’s survival; his claim doesn’t include what will
happen if deforestation merely slows down.

• Any notion that can co-exist with a claim without contradicting it can be said to be
“consistent” with the claim. That’s not to say that it proves or even supports the
claim, just that it doesn’t go against the logic of it. Conversely, something that’s not
consistent with a particular claim is something that contradicts that claim.

• You’re asked to measure each choice against two statements. The best statement to
begin with is probably the politician’s statement because a direct contradiction of an
argument is usually easier to spot than a statement that’s merely consistent with an
argument. As soon as you find a statement doesn’t contradict the politician, move on.

• It is important that you notice the difference in the character of the two claims. The
biologist makes a statement of necessary cause: “it is necessary to slow deforestation
if the koala is not to approach extinction.” The politician makes a statement of
sufficient cause: “to save the koala from extinction, it is sufficient (that is, enough —
nothing else is needed) to stop deforestation.” The concept of necessary vs.
sufficient conditions is discussed at length in Unit 3 and appears elsewhere
throughout the Kaplan course.

12. (A)
Sometimes, reading the question stem doesn’t tell you what you need to know in order to
direct your attack on the stimulus. In such a situation, it’s a good idea to preview the
answer choices to get a better idea of what you’ll need to accomplish. In this case, the
question stem reads “In the passage, the author” which isn’t particularly helpful. So this is
one of those rare times when a quick look at the answer choices is advisable.

Each choice describes a method of argument, which tells us that despite the ambiguous
question stem, we should approach the stimulus the same way we would approach any
other method of argument question. This means we should focus on how the evidence is
related to the author’s conclusion. The author claims that it is premature to conclude that
telepathy exists, since there is no clear evidence to the contrary. Even though some research
might suggest that telepathy exists, that research can be explained away. In other words, the
author basically argues that telepathy doesn’t exist because no one can yet prove that it
does. This is the method, described in more general terms, found in (A).

32 © K A PL A N
LSAT PREP ________________________________________________________________ LSAT Test II Explained: Section II

(B) What particular experiments?

(C) There is no specific piece of evidence mentioned.

(D) The author concludes that it is too early to conclude that telepathy exists, based on a
complete lack of evidence to the contrary. This may not be the most convincing argument
in the world, but the evidence as stated does support the conclusion.

(E) As mentioned in (D), the author does cite evidence to support her position, namely, the
lack of evidence to the contrary.

• If the question stem doesn’t tell you enough about your task, quickly preview the
answer choices. They’ll help you get a better sense of what you will need to
accomplish.

13. (C)
The stem tells us that in addition to the information in the passage, we’re to assume that
salaries have kept pace with rising prices this year. To see what effect this has, let’s go to the
place in the stimulus where salaries are mentioned. The last sentence says that if prices
have risen beyond a level most people can afford, then it must be a fact that salaries have
not kept pace with rising prices. Therefore, if salaries have kept pace with rising prices (as
given in the stem), then we can infer that prices have not risen beyond a level most people
can afford—that’s choice (C).

(A) As we’ve seen above, we can infer that prices have not risen beyond a level most
people can afford. According to the first sentence, that would mean that if we know retail
stores experience a decrease in revenues, we can infer (A), that attitudes have changed.
However, we’re told nothing about a decrease in revenues, and we thus have no grounds
for inferring (A).

(B) We know that if retail stores experience a decrease in revenues, then one of two things
must have happened. We know that one of the things (prices rising beyond an affordable
level) hasn’t happened, but we don’t know that the other thing (change in attitudes) hasn’t
happened. Therefore, we have no grounds for concluding that the decrease in revenues
doesn’t occur.

(D) Because we can infer that prices haven’t risen beyond an affordable level, we can also
infer that if attitudes toward extravagant gift giving haven’t changed, the store won’t
experience a decrease in revenue this season. However, we have no grounds for saying that
the attitudes haven’t changed.

(E) It’s possible that neither of these two things has happened (as a matter of fact, we know
the second thing hasn’t happened). We know only that if neither thing has happened, retail
stores won’t experience a decrease in revenue.

© K A PL A N 33
LSAT PREP ________________________________________________________________ LSAT Test II Explained: Section II

• In order to understand the structure of a long chain of formal logic statements, it


might help to circle the key words such as “if,” “then,” “either,” and “or.”

• Don’t be intimidated by a long chain of reasoning. Often, in stimuli involving formal


logic statements, the answer will come from the correct interpretation of a single
statement. To save time, go right to the place relevant to the information in the
question stem, and work from there. Here, the right answer can be found by
combining the new “salaries” information in the stem with the last sentence. The rest
of the passage is used as fodder for the wrong choices.

• The new information in the stem combined with correct choice (C) simply forms the
contrapositive of the last sentence. The last sentence: “if prices have risen . . . then
salaries have not kept pace . . .” The contrapositive: “if salaries have kept pace (i.e.,
NOT-not kept pace) . . . then prices have not risen . . .” To form the contrapositive of
an if-then statement, reverse and negate the terms. If one of the terms is already
negative, negating it makes it positive (NOT-not kept pace = have kept pace).

14. (C)
Suicide rates in October and November of 1929: tidal wave or a ripple in a kiddie pool?
The author certainly believes that the so-called “suicide wave” following the stock market
crash in October 1929 was not such a big deal. In support, the author notes that the suicide
rates in October and November of 1929 were lower than the rates for the months before the
crash. We need to weaken that argument, meaning that we are looking for a choice
suggesting that suicide rates in those months were in fact unusually high. The whole key
here is: high as compared to what? We know October and November were no big deal
compared to the other months in 1929, but maybe a better comparison would be with the
suicide rates in Octobers and Novembers of the surrounding years. This makes for a better
comparison because it eliminates other seasonal factors that might come into play when
analyzing suicide rates throughout the year. Choice (C) picks up this distinction. When
viewed in comparison with other October-November periods, 1929 does in fact seem like a
high suicide period. Thus (C) weakens the argument.

(A) is out of the scope. The issue is whether the suicide rates were unusually high during
the period mentioned and not the root causes of suicide in general.

(B) and (D) strengthen the argument. If October and November (or late months in general)
typically have high suicide rates as compared to other months, then lower rates for those
months in 1929 would support the author’s view that the “suicide wave” was in fact more
myth than reality.

(E) doesn’t affect the argument either way: knowing that suicide rates are expected to be
different in different seasons doesn’t tell us whether the results in 1929 were unusually high
or low.

34 © K A PL A N
LSAT PREP ________________________________________________________________ LSAT Test II Explained: Section II

• Always paraphrase to make the argument yours. This question takes many twists
and turns, and if you don’t put the argument into your own words, it is easy to lose
sight of your task.

• Many LSAT questions require you to make comparisons, and you’ll need to focus
on the scope of the argument to tell whether the comparison is relevant to the
conclusion. In this case, the issue is whether October and November of 1929 has
unusually high suicide rates, and the best way to answer that question is probably to
compare these rates to the Octobers and Novembers of other years—not to compare
them to the other months of 1929.

15. (B)
The stimulus gives a general rule relating to a sports figure: she doesn’t combine publicity
tours and playing tours—i.e., they’re mutually exclusive alternatives. If she visits a city to
play in a big competition she doesn’t do publicity in bookstores, and vice versa. Then we
get a specific case where the rule is applied: she’s visiting London to play in a major
competition; therefore she’s not doing any bookstore publicity. (B) follows the same
pattern. First, a rule relating to a hospital’s emergency room: they only do minor work
when they have no emergencies to attend to. Again, these are mutually exclusive
alternatives: when they have emergencies, they don’t do minor work, and if they do minor
work, they can’t have emergencies. (B) then relates this rule to the situation on Monday
night. They were doing minor work that night so therefore they did not have any
emergencies to attend to.

(A) doesn’t deal with mutually exclusive alternatives. Moreover, (A) switches from saying
“many” wasps in the evidence, to “any” (meaning “all”) wasps in the conclusion, a
problem the stimulus doesn’t have.

(C) is the only half-way decent wrong choice. You may have read the first sentence as
stating mutually exclusive alternatives: either the summers are hot or tomatoes don’t
thrive. The problem is two-fold: first, the alternatives are not stated plainly, as they are in
the stimulus and correct choice (B), and second, even more importantly, the conclusion
introduces a note of uncertainty, “probably,” which is not found in the stimulus.

(D) is a simple chain of reasoning: X leads to Y; Z leads to X; Therefore, Z leads to Y. Not at


all similar to the stimulus.

(E) Again, no alternatives here. We’re just told that since the blade that murdered Q has a
certain characteristic, and butter knives also have that characteristic, poor old Q might
have gotten it with a butter knife.

© K A PL A N 35
LSAT PREP ________________________________________________________________ LSAT Test II Explained: Section II

• On a long Parallel Reasoning stimulus like this, the Big Picture is more important
than the details. If you just grasp the central idea, “two alternatives that can’t both be
true at the same time; one is true, so the other isn’t,” you can make short work of the
wrong choices.

• Two of the choices, (C) and (E), draw tentative conclusions; they speak of what is
“probably” the case, or what “may have been” the case. This is not a mere detail;
when the stimulus draws a definite, unqualified conclusion, there is no way that an
argument drawing a tentative conclusion can be parallel.

16. (E)
We’re told that, from 1950 to 1980, the percentage of all ski-related injuries at ski resorts that
did not occur on the slopes increased from 10% to 25%. That must mean that the percentage
of ski-related injuries at ski resorts that did occur on the slopes must have decreased over
that same period. Think about it this way: there are only two possible alternatives for a ski-
related injury: either it occurs on the slopes or it does not occur on the slopes. So if the
share of ski-related injuries represented by one type of injury goes up, the share
represented by the other type MUST go down. That’s choice (E).

(A) We have no information about the number of the different types of ski-related injuries
occurring, only about the percentage of total injuries represented by the two different types
of injuries. Since we don’t know how or whether the total number of injuries changed, we
can’t infer (A).

(B) We can infer nothing about the amount of alcohol consumed. It’s true that increased
alcohol consumption causes an increase in injuries off the slopes, but we don’t know that
those injuries have increased numerically, nor do we know that alcohol consumption is the
only thing that could bring about an increase.

(C) Although the stimulus says improved ski boots and bindings have decreased the
incidence of injuries on the slopes, it never implies that they have any effect on off-slope
injuries.

(D) is unwarranted for two reasons: first, the fact that better boots and bindings have
reduced the incidence of on-slope injuries in the past is no guarantee that they will do so in
the future and, second, (D) speaks of “ski-related injuries” in general, and we have no
reason to believe that boots and bindings have any effect on the incidence of off-slope
injuries.

36 © K A PL A N
LSAT PREP ________________________________________________________________ LSAT Test II Explained: Section II

• When you first read this stimulus, you may have been tempted to try to work with
the numbers. Don’t do any more work than you have to; this is not a math test.
Rather than trying to pre-phrase an inference on a complex mathematical stimulus
like this, go to the choices; use them to tell you what kind of inference you want.

• It is not uncommon on a large stimulus like this for the correct answer to be a
“small” inference, based on one statement, rather than a “big” inference, linking all
the statements together.

• Remember the “numbers vs. percentages” flaw? It is discussed as one of the logical
flaws in Unit 3. Sometimes, the testmakers will throw this form of erroneous
reasoning into a stimulus and ask you to identify the flaw. Here, there’s no flaw in
the stimulus’ reasoning per se, but choice (A) is wrong on account of substituting
numbers for the passage’s percentages. Thus, understanding the difference between
raw numbers and percentages can lead you right to the correct choice, or in other
cases, help you to narrow the choices down.

17. (B)
We’re told that the incidence of on-slope injuries decreased from 9 injuries per 1000 skiers in
1950, to 3 injuries per 1000 skiers in 1980. That means a skier was much less likely to suffer
an on-slope injury in 1980 than in 1950. Clearly, (B) is in direct contradiction with this.

(A) Although we know the “incidence” or rate of injuries on the slopes definitely
decreased over this period, we don’t know anything about the actual number of injuries. If
there were many more skiers on the slopes in 1980 (at least three times as many), then it’s
quite possible that answer choice (A) is correct and on-slope injuries did go up.

(C) The stimulus says absolutely nothing about the reporting of injuries, so it’s not
contradicted by (C).

(D) The stimulus provides us with absolutely no grounds for concluding anything about
the actual number of skiers; we’re only told about the number of injuries per 1000 skiers,
without being told the necessary information: how many thousand skiers there actually
were. So answer choice (D) may or may not be true; it doesn’t contradict any information
we’re given.

(E) Since we’re told that 25% of all ski related injuries occurred off the slopes, and that
those injuries include things like “falling down steps,” (E) is actually a safe inference based
on the stimulus.

• Even though this stimulus looked difficult, the questions based on it were actually
rather easy—especially this second question. It’s worth taking a good crack at a
double-question stimulus, even if it looks very difficult. One of the questions is
often easy, and sometimes both are.

© K A PL A N 37
LSAT PREP ________________________________________________________________ LSAT Test II Explained: Section II

18. (B)
Is nesting experience important for the breeding success of birds? The author uses the
research of Dr. Snow to suggest that nesting experience itself must play a role in the
increased breeding success of blackbirds. First-timers may not be as successful at breeding
as older birds, but is nesting experience itself the only possible explanation for the
difference? If another attribute related to reproduction, such as the ability to lay viable
eggs (as in (B)), improved with successive nesting experience, then the author’s account of
the difference becomes much less convincing.

(A) contains an irrelevant comparison. The issue is whether blackbirds benefit from nesting
experience, and not whether they are better or worse than other birds at nest-building.

(C) extends the scope from blackbirds to birds in general. Still, this information would
strengthen the argument by bolstering the connection between breeding success and
nesting experience, which indeed is the crux of the conclusion in the last sentence.

(D) supports the author’s evidence in the next-to-last sentence, and may even therefore
slightly support the author’s claim that something else besides size is responsible for
blackbird breeding success. Whatever small strengthening effect (D) has, it certainly
doesn’t weaken the argument at all.

(E) is irrelevant, since the percentage of birds that survive long enough to nest is
unconnected to the issue of what factors influence the breeding success of those birds that
do nest and reproduce.

• Be on the lookout for alternative explanations—other ways to explain the facts in the
passage. An alternative explanation, if true, can weaken an argument. Also, ruling
out a reasonable alternative explanation can strengthen an argument (we saw a good
example of this in Q. 10 on this section).

• Look to aggressively eliminate choices when you can’t find a choice that matches
your pre-phrase (or when you cannot form a pre-phrase). In this case, it may be
difficult to spot exactly why (B) weakens the argument, but you should be able to
eliminate the other four answer choices which conform nicely to the wrong answer
patterns you learned (or will learn) in class.

19. (D)
This question was built for denial—the Kaplan Denial Test, that is. The author believes that
if airlines want to cut down on pilot errors, they should change their training to include
more actual flying time. However, what if (D) is false, and the lack of flying time is not an
important factor in plane crashes? In that case, it would seem pretty silly to emphasize
flying time in training. So, if (D) is false, the author’s entire argument would fall apart,
which means that (D) must be a necessary assumption.

38 © K A PL A N
LSAT PREP ________________________________________________________________ LSAT Test II Explained: Section II

(A) Use the Denial Test to eliminate wrong answers, too. Suppose that training programs
cannot completely eliminate pilot errors. Could training programs with beefed-up actual
flight time still be a good idea? Sure, and so (A) is not necessary to the argument.

(B) is irrelevant. The author makes a claim about how airlines ought to design their
training. What airlines actually do regarding “additional training” has no bearing on this
point.

(C) doesn’t address the author’s argument, which concerns the relationship of actual flying
time to pilot error. The relationship of pilot error to crashes is another question.

(E) Suppose that communication skills are important. Does that mean that they are more
important than flight time, or that training should not be re-designed to emphasize flight
time? No, and so (E) is not necessary, either.

• Questions like this demonstrate the effectiveness of the Kaplan Denial Test. Use it
both to confirm correct answers and to eliminate wrong answers.

20. (C)
This stimulus lends itself to symbols. All savings accounts are interest-bearing accounts,
“All X are Y.” The interest from some interest-bearing accounts is tax free, which translates
into “Some interest-bearing accounts have tax-free interest,” or “Some Y are Z.” So some
savings accounts have tax-free interest, “Therefore, some X are Z.” We see the same pattern
in (C). All great photographers are artists, “All X are Y.” Some artists are intellectuals,
“Some Y are Z.” Therefore, some great photographers are intellectuals, “Therefore, some X
are Z.” A perfect match.

(A) All artists are intellectuals, “All X are Y.” Some great photographers are artists, “Some
Z are X.” Already that goes off the rails; you can move on immediately.

(B) All great photographers are artists, “All X are Y.” All artists are intellectuals, “All Y are
Z.” The stimulus only had one “all” statement, so you know that answer choice (B) isn’t
parallel.

(D) All great photographers are artists, “All X are Y.” Some great photographers are
intellectuals, “Some X are Z.” Again, (D) is no good before you even get to the conclusion
(“Therefore, some Y are Z”).

(E) All great photographers are artists, “All X are Y.” No artists are intellectuals, “No Y are
Z.” This is the opposite of what we need to parallel the second statement in the stimulus.
Read no further.

© K A PL A N 39
LSAT PREP ________________________________________________________________ LSAT Test II Explained: Section II

• A word of warning: don’t get mixed up in your symbolization between one choice
and another. There’s a special danger of that here, because all the choices use the
same terms (artists, intellectuals, photographers). Start fresh with the symbols with
each choice (All X are Y). For (A), X is artists, for (B), X is photographers. Stay
consistent within each choice, but don’t confuse choices.

• This Parallel Reasoning question is perfectly algebraic. Therefore, you can approach
it almost like a Logic Games question, in that once you get the answer, there’s no
reason to look at the other choices—at most, give them a quick glance if you feel you
must double-check.

21. (D)
First you get a general rule: the more genetically similar two species are to one another, the
more recently they diverged from a common ancestor. Then we get a specific case study—
some conclusions based on this rule. Scientists figure that first bears and raccoons
diverged, then red pandas diverged from raccoons, then giant pandas diverged from
bears. What can we infer? Well, the scientists concluded that giant pandas diverged from
bears more recently than they diverged from raccoons (i.e. they diverged from raccoons
along with all the other bears before they diverged from the other bears). According to the
rule, therefore, scientists must have observed (D), the fact that giant pandas are more
genetically similar to bears than they are to raccoons.

(A) We have no idea what scientists “originally thought” about the genetic relationship
between giant pandas and raccoons, so we can’t infer (A).

(B) We’re told that giant pandas “diverged” from other bears millions of years ago; that
gives us no reason to infer that scientists therefore consider them a species of bear.

(C) The stimulus speaks of divergences occurring “a few million years later” and “30 to 50
million years ago”; that certainly does not qualify as establishing the timing “within a few
years.”

(E) Since bears (including the ancestors of giant pandas) diverged from raccoons a few
million years before red pandas did, we would expect red pandas to be more closely related
to raccoons genetically than are giant pandas.

• Always try to grasp the structure of the stimulus. Here, there’s a rule, followed by an
application of the rule. You must understand the rule, so you can understand the
point of the facts you’re given; otherwise, you’ll become lost trying to wade through a
mass of apparently trivial facts.

40 © K A PL A N
LSAT PREP ________________________________________________________________ LSAT Test II Explained: Section II

22. (A)
What is responsible for the increased death rate for asthma over the last decade? The
author mentions three potential explanations. The first two—increased, more accurate
reporting and an increased urban population—cannot, according to the author, explain
why asthma death rates have increased dramatically in cities with long-standing accurate
records and very small urban populations. From this, the author makes the remarkable
conclusion that the increase must be due to a third explanation: the use of bronchial
inhalers. This argument needs a lot of help, and fortunately, four of the five answer choices
are intended to provide support, which means to help fill in the gaps. (A), however, is
irrelevant: the only urban factor mentioned is increased pollution, and what relation a
doubling urban population has to pollution is ambiguous at best. Mostly, however, (A)
simply has no bearing on the inhaler question or the first two factors mentioned and then
discredited as causes for the phenomenon. Each of the other choices, as we’ll discuss
below, does have a direct bearing on these issues, whereas (A) does not.

(B) strengthens the argument by helping to rule out increased reporting as an alternative
explanation for the increase in the asthma death rate. If records have been just as good for
the past twenty years, then an increase in deaths over the past ten years cannot be
influenced by better reporting.

(C) and (D) both suggest qualities of the bronchial inhalers that indicate that their use
might play a role in increased asthma death rates. It is plausible that irritated lungs and
avoiding medical treatment are causally related to death.

(E) If bronchial inhalers had been available the entire time, it would be hard to believe that
they could suddenly be responsible for the recent increase in the asthma death rate. (E)
thus strengthens the argument by suggesting a connection between the recent doubling of
the death rates and the introduction of the proposed cause, the inhalers.

• Reading the question stem first tells us that four of the choices will strengthen the
argument and one will not, which suggests that this may be a time-consuming
question best saved for late in the section. As it turns out, this question appears late
in the section, but you just as easily could have encountered it early on, in which
case this Section Management strategy would take on greater importance.

© K A PL A N 41
LSAT PREP ________________________________________________________________ LSAT Test II Explained: Section II

23. (E)
Now we’re asked to identify a necessary assumption of the same argument we tackled in
22. In the stimulus, the author presented two possible explanations for the increase in the
asthma death rate, and then jumped to the conclusion that the increase must be due to a
third, previously unmentioned reason, the dreaded bronchial inhalers. For this conclusion
to be valid, improved recording, an increase in urban pollution, and bronchial inhalers
would have to be the only possible explanations for the increase. This is choice (E). If there
were more potential explanations (the denial of (E)), then the rejection of the first two
would still leave us with multiple candidates, and it would be premature to point the
finger at those nefarious bronchial inhalers.

(A) All the author says about urban pollution is that it has increased; how big of an
increase has taken place we don’t know. It certainly could have doubled without the
argument falling apart. In other words, the author is not relying on the fact that it hasn’t
doubled, and thus is not assuming (A).

(B), if true, would weaken the argument by suggesting another possible explanation for an
increase in the death rate.

(C), if true, would probably strengthen the argument, but is not necessary to the argument,
which could still be true if bronchial inhalers were only “unsafe” when the
recommendations were ignored (the denial of (C)).

(D), which focuses on deaths ultimately resulting from other diseases besides asthma, is
irrelevant to an argument based on asthma death rates, “the death rate from this disease,” as
the stimulus puts it.

• Why does the Kaplan Denial Test work? Think of it this way: if you needed to know
whether a certain part of your car was necessary for it to run, how could you test it?
One way would be to remove the part, and see what happens. If the car can still run,
then the part wasn’t necessary. On the other hand, if the car falls apart and doesn’t
run anymore, then it would be fair to infer that the part was necessary after all. Don’t
try this at home.

• Any choice that is out of the scope or irrelevant in some way to an argument cannot,
by definition, be an assumption on which that argument depends.

42 © K A PL A N
LSAT PREP ________________________________________________________________ LSAT Test II Explained: Section II

24. (E)
The author in this one doesn’t think much of the political sagacity of artists. Still, she’s
careful not to issue a blanket condemnation. She says there is “little” point in looking to
artists for political insight, not no point. She says “when taken as a whole” artists’
statements indicate that artistic talent and political insight are “rarely” found together, not
never found together. And, most important to us, she says that “most” artists have political
views that are less insightful than the views of reasonably well-educated people who aren’t
artists, not that all artists have such views. Well, if she doesn’t hold that all artists have less
insightful political views than any reasonably well-educated person, she must hold that
some artists do hold political views that are just as insightful as the views of some
reasonably well-educated person. Thus, based on the passage, we can infer (E).

(A) is too strong; as we’ve seen, the author is careful not to say that absolutely no artists
have political insight.

(B) The author compares artists to “any reasonably well-educated person” but never says
that art alone could make a person reasonably well-educated; indeed, the author is more
interested in drawing a contrast between artists and “reasonably well-educated” people.

(C) is again too extreme. The author only spoke of “most” artists falling short of any
reasonably well-educated person in terms of political insight, which implies that some
artists do indeed make the grade.

(D) involves a scope shift. The author spoke of the rarity of finding political insight and
artistic talent together. Now, having political insight is not the same thing as being a
politician. It may be true that plenty of politicians lack political insight (which implies
some profound understanding of the issues); in fact, I’m sure you could think of many
examples. These uninsightful politicians might have loads of artistic talent. So (D) can’t be
inferred.

• The inference here is unusual; it requires that you understand the tone of the
stimulus. In effect, you’re asked to make an inference based on what the author does
not claim: she doesn’t claim that all artists are political simpletons, so she’s
effectively admitting that some artists aren’t.

• Because of the unusual nature of the correct inference, you mightn’t have recognized
it right away. However, you could still get it by elimination, because the wrong
choices all go off the rails in familiar, definite ways: too extreme, (A) and (C); making
a false comparison or distorting a comparison made in the stimulus, (B); scope shift
by changing terms, (D). When you know the forms of the common wrong choices,
you can eliminate them quickly.

© K A PL A N 43
SECTION III:
LOGIC GAMES

44 © K A PL A N
LSAT PREP ________________________________________________________________ LSAT Test II Explained: Section III

GAME 1 — Mammoth Hirings


(Q. 1-5)

The Action: In no time at all you should see that this game asks you to arrange the entities
in order of hiring—a sequence game. Moreover, it’s a special type of sequencing game, one
we call a “free-floating” sequencing game because all of the entities are described relative to
each other, as opposed to in a standard sequencing game in which the entities are slated for
definite slots (Monday to Friday, 1 to 7 in a line, etc.). In contrast, in “free-floating” games,
the important aspect is where each entity is relative to the others—Haga is before Duvall,
Duvall is after Irving, etc. On the basis of these types of rules, there are no specific slots
we’re asked to fill in, although sometimes, as in this case, we’re able to deduce much of the
relative ordering. That is, in easy games of this type (and this is one of the easier ones), we
get so much relative information that we’re able to fit entities into specific time slots, but
that’s not always the case.

Each of the nine new workers—Brandt, Calva, Duvall, Eberle, Fu, Garcia, Haga, Irving, and
Jessup—was recently hired. The Key Issues will be:

1) What worker can, must, or cannot have been hired before or after what other worker?
2) What worker can, must, or cannot be hired on the same day as what other worker?

The Initial Setup: Many people find it helpful to visualize a “free-floating” sequencing
game in a vertical fashion. Let’s put the workers who are hired earlier above the workers
who were hired later. Remember to list the workers’ letters off to the side, and we’re ready
to hit the rules:

BCDEFGHIJ

The Rules:

A good way to start is to find a worker or workers who might come at the top of the sketch;
that is, anyone who doesn’t seem to be hired after other workers. If you scan the rules,
you’ll see that H fits that description, as does the FI pair from the first rule. Let’s therefore
start by putting an “FI” and an “H” at the top of the sketch.

© K A PL A N 45
LSAT PREP ________________________________________________________________ LSAT Test II Explained: Section III

1) F and I were the only two workers hired on their day, so whatever is true of one is true
of the other. That means they’re an inseparable pair. We’ve already put our “FI” pair at the
top of the sketch, so let’s move on.

Where to now? We may as well find entities related to the ones we’ve already put on the
page. Rule 6, containing Irving, links up nicely:

6) Break this information down into parts. First of all, D was hired after I. Draw a line
down from the “FI” pair to a “D.” Secondly, D was hired before E, so draw a line down
from the “D” and add an “E.”

5) We now continue to incorporate the rules that link up with the entities already on the
page: “H was hired before D,” so we can draw another line up from D connecting to the H
we drew earlier.

4) Now we can work down from E: This rule says that E was hired before B, so draw a line
down from the “E,” and add a “B.”

8) This seems like the logical rule to incorporate next: B was hired before J, so draw a line
down from the “B,” and add a “J.”

7) Now that J and B are placed, we can deal with Rule 7: G was hired after J and B. Add a
“G” below the “J.”

2) Like F and I, C and G were hired on the same day (along with no one else that day), so
add C to the G we’ve just placed below J.

3) We’re left with this one general rule: Except for the two pairs of workers in Rules 1 and 2
(FI and CG), on every other day exactly one employee was hired. Basically a loophole
closer, no need to draw anything. (Note that this does confirm a total of seven hiring days,
although they need not be consecutive.)

Key Deductions: Much should be clear just by reviewing the master sketch: H and FI
must have been hired on days 1 and 2, in either order. D must have been hired on day 3;
E on day 4; B on day 5; J on day 6; and GC on the last day, day 7. All that’s ambiguous, in
fact, is the H vs. FI relationship. Talk about concrete information! Small wonder that there
are so many “non-if” questions.

46 © K A PL A N
LSAT PREP ________________________________________________________________ LSAT Test II Explained: Section III

The Final Visualization: And here is that neat, accessible sketch:

FI H

CG
The Big Picture:

• In a “free-floating” sequence game, you have to start somewhere, and that means
getting at least one or more entities down on the page. A logical place to start is at the
top, so aggressively search the rules for those entities that seem above the others, and
get those down on the page. Don’t worry if you make a mistake—you can always
revise the sketch as you focus more exhaustively on each rule. But staring at the page
afraid to make the first move will never do you any good.

• Attack the rules in whatever order is most efficient. Usually, we’ll just work through
the rules in the order given, if there seems to be no advantage in doing otherwise.
But be ready to abandon this regularity when the situation warrants. The LSAT
rewards proactive thinkers, not robotic muddlers.

• In all games, the setup absolutely requires careful thought—and that takes time. A
careless master sketch can turn a straightforward game into a nightmare. It may have
taken a few minutes to piece this puzzle together, but look at the result: Our nearly-
resolved sketch allows us to practically read the answers right off the page.

• Always break a two-part rule (like Rules 6 and 7) down and deal with each part
separately.

© K A PL A N 47
LSAT PREP ________________________________________________________________ LSAT Test II Explained: Section III

The Questions:

1. (D)
As long as you constructed your sketch carefully, this question is a gimme. The master
sketch clearly shows that the last two workers hired were Garcia and Calva, choice (D).

• After a lengthy setup, the first question will often be quite straightforward. This is
the testmakers’ way of rewarding those who took the time up front to carefully work
through the setup.

2. (A)
Again, if your sketch is complete and accurate, you’re rewarded with another ten-second
question. The FI pair and H were hired on days 1 and 2 (although we don’t know in which
order), D was hired on day 3, and E on day 4, choice (A).

• Trust the work you put in up front—the reason you put in this time is to save time
later on questions like this. If you thought “this is too easy, I’d better check my work
to make sure,” you’re defeating the purpose of milking the setup for all it’s worth.
Confidence is key on all LSAT sections, and especially on Logic Games.

3. (A)
Here you’re looking for the workers who must have been hired before J. Again, let your
master sketch do the work for you. Just count the entities who must have come before J: F,
I, H, D, E, and B, which makes a total of six, choice (A).

• And that makes three questions that you can race through as long as you took the
proper amount of time up front. And there’s another benefit: By working this way,
you aren’t just getting quick points—you’re amassing a lot of extra time to use on
later, more complicated games.

4. (E)
Thanks to the master sketch, you know nearly everything about the entities. It’s an easy
(and quick) matter of checking each choice looking for the one that must be true.

(A) No, D must have been fourth, not first.

(B) and (C) H could have been hired first, but so could the FI pair. Each of these choices is
possible only. One of ‘em is true, but we can’t figure out which.

(D) No—clearly the first two workers hired could be Irving and Fu, with Haga third. (D)
therefore need not be true.

48 © K A PL A N
LSAT PREP ________________________________________________________________ LSAT Test II Explained: Section III

(E) is the only choice left, but for the record: The sketch clearly shows something we’ve
known from the very beginning: that either H was hired first, or F and I were. (E) must be
true and is the answer.

• Some would argue that (D) CANNOT be true, while others make a case that it can
based on an ambiguity inherent in Rule 1: F and I are hired on the same day, but isn’t
it still possible that one is hired before the other on that day? If so, then H could be
first, and then F second and I third, even though F and I were both hired on the
second day. This is a subtle logistical ambiguity; the solution is that the testmakers
steer clear of the issue altogether. It’s easy to eliminate (D) based on the reasoning
cited above—no matter how we interpret this issue, clearly there’s a simple way to
show that (D) need not be true. The point is this: Don’t get bogged down in logical
intricacies that need not play a part in answering a question—seek the easiest way
possible to eliminate choices and to get to the credited answer.

• On test day, you probably wouldn’t have bothered to check choice (E). After all of
the other choices have been eliminated, what’s left must be the answer. Don’t waste
time double checking what you know must be the right answer.

5. (D)
If E was hired on Monday, the earliest day on which B could have been hired is Tuesday. If
B was hired on Tuesday, the earliest J could have been hired is Wednesday. And if J was
hired on Wednesday, the earliest G (as well as C, of course) could have been hired is
Thursday, answer choice (D).

• Don’t let a slight aberration in a question stem throw you. Even though we hadn’t
previously been faced with “Monday” or any other day of the week, all that’s
necessary to get the answer is to stay focused and work systematically.

• Section Management is crucial for Logic Games success! Always search for the
easiest game to begin your work on the LG section. Here, this easy game happened to
come first, but on other tests it may have come later on. Preview the section and
begin with the game that’s easiest for you, no matter where that game appears. Doing
so will not only get you a few solid points right off the bat, but will also increase
your confidence which can only bode well for you on the rest of the section.

© K A PL A N 49
LSAT PREP ________________________________________________________________ LSAT Test II Explained: Section III

GAME 2 — Five Floors, Eight Apts.


(Q. 6-12)

The Action: After conducting a thorough overview of this game, you’ll find our task once
again is that of sequencing—this time, ordering eight people on five floors of an apartment
building. The one small twist is that in order to fit the eight people, some floors must get
more than one person. No problem: We’re told that each floor has one or two apartments,
and that each person lives in a separate apartment. Mainly, we’ll be asked to put the
entities in order based on which floor they live on. The Key Issues in this game are:

1) Which floor does each person live on?


2) Which people can, must, or cannot live on the same floor?

Another issue lurks below the surface and will help us to resolve the major key issues
listed above, namely: How many apartments are on each floor? Think about this for now—
we’ll come back to it in our discussion of Key Deductions.

The Initial Setup: A natural way to represent five floors is the way you’d probably sketch
it in real life: List the numbers vertically from 5 at the top to 1 at the bottom, exactly the
way we typically envision floors of a building. Remember to list the entities off to the side,
and you’re all set:

JKLMNOPQ
5

The Rules:

1) So J must live on one of the floors with two apartments. How you symbolize this is up to
you, but make sure that it’s not misleading. “J __” is a good way to keep this rule in mind.

50 © K A PL A N
LSAT PREP ________________________________________________________________ LSAT Test II Explained: Section III

2) K lives on the floor directly above P. Which floors? Dunno, but K should do it for a
visual cue. P

3) The second floor has only one apartment. Build this rule directly into the master sketch.

4) M and N must live on the same floor. Wherever you have one, the other will be there as
well. “Always MN” is one way to symbolize this.

5) is the flip side of Rule 4. O and Q will never be on the same floor. “Never OQ” should do
it as a visual reminder.

6) Don’t be so quick to symbolize any rule; think about it first. L must be alone on a floor,
and the floor that L is on must have only one apartment. Add this information to your
sketch.

7) Q doesn’t live on the first or second floors, meaning that Q does live on floor 3, 4, or 5.
Build this directly into the sketch.

Key Deductions: We alluded above to an important sub-issue regarding the number of


apartments on each floor. This is key: With eight residents, eight apartments, and a
maximum of two residents per floor, it follows that this building must have three floors
with exactly two tenants each, and two floors with exactly one tenant each. Check it out
for yourself if you need confirmation. Moreover, we know two of our three resident pairs:
M/N (Rule 4) and J plus someone (Rule 1). Knowing all this in advance should at least help
you get a handle on ordering the entities.

What else? Right from Rule 2 we can deduce that K can’t live on the first floor and P can’t
live on the fifth floor (the latter of which gets the point for Q. 7).

What else? Well, we know a lot about floor 2, so let’s look into that a little further: Floor 2
has only one apartment (Rule 3), so you can deduce that M and N, who live together on the
same floor (Rule 4), can’t occupy the second floor (thank you, Question 6). J, who must
live on a floor with two apartments (Rule 1) can’t occupy the second floor, either. That
makes three entities who cannot live on the second floor—four, when you remember
indented Rule 7. And be sure and take the extra step to turn this negative information to
the positive: K, L, O, and P are the only possibilities for floor 2.

Remember, too, that we have only two “solo” residents: L, and one other. Everyone else
lives on a floor with someone else. And notice: L could occupy that one-resident floor 2, but
need not.

© K A PL A N 51
LSAT PREP ________________________________________________________________ LSAT Test II Explained: Section III

The Final Visualization: Here’s all you have going into the questions:

JKLMNOPQ no P 5 J __

Q on 3, 4, or 5 4 K
P
3
Always MN
(K L O or P) 2 __ ||
Never OQ
no Q, no K 1
L

The Big Picture:

• The numbers aspect of Logic Games is always important. Keep asking yourself,
“How can the entities fulfill the given requirements?” The numbers in this game
give you a very helpful foundation as you go into the questions.

• Don’t be intimidated by a large number of rules—the more information you’re given,


the more you have to work with and the better you’ll be able to impose restrictions
on the entities. Concrete information leads to points; abstract information often leads
to confusion. It’s the games with few rules, and thus much ambiguity, that are
generally the tougher challenges.

• Technically, the game should have spelled out that the floors are numbered 1
through 5 from bottom to top rather than vice versa. (If, contrary to everyday practice,
you put floor 1 on top, the game won’t work; doing this screws up the K/P rule.) It’s a
reminder that LSAT Logic Games are meant to reflect everyday life assumptions.
When in doubt, go with the simplest, most everyday, most straightforward
interpretation. The testmakers have enough weapons in their arsenal; they don’t try
to trick us by basing games on upside down buildings or any other counter-intuitive
design.

52 © K A PL A N
LSAT PREP ________________________________________________________________ LSAT Test II Explained: Section III

The Questions:

6. (D)
Off to a fast start: We deduced above that the M/N pair can’t live on the second floor, so
(D) is the answer. How did we deduce this? Simply by combining Rules 3 and 4: M and N
are together, but 2 is a solo floor. Our master sketch confirms that the only entities that can
live on floor 2 are K, L, O, and P.

• A must be true/non-if question—one that comes with no new information—is almost


always testing for a deduction built into the rules. It doesn’t necessarily have to be a
huge deduction. Figuring out in advance which entities can and cannot go on the
second floor may not crack this game wide open, but it does lead to a very easy first
point.

7. (E)
What CANNOT be true? We deduced a whole bunch of things that fit this description,
most notably regarding floor 2: J, M, N, and Q can’t live there. Scan the choices . . . no luck.
Well, we also deduced from Rule 2 that K can’t live on the first floor and P can’t live on the
fifth floor. The latter is cited in correct choice (E).

• This game is fairly wide open, yet the first two questions have nevertheless been
“non-ifs,” testing for things that can be deduced solely from the setup. All of this
reconfirms the usefulness of milking the setup and rules for as much information as
possible, before proceeding into the questions.

8. (A)
Break apart this stem and take it one piece of information at a time. J occupies the fourth
floor with one other person (Rule 1). You should create a new sketch next to Q. 8, and insert
J into floor 4. We’re also told that K is on the fifth floor—insert that, too—which means that
P must join J on the fourth floor (Rule 2). M and N must occupy the same floor (Rule 4),
and the second floor has only one apartment (Rule 3), so M and N must take either the third
or first floor (they’re the only floors left with two empty spaces). The other entities will
split up the remaining floors, provided that Q and O stay split up (Rule 5) and L occupies
a floor alone (Rule 6). On to the choices:

(A) Can O live on the first floor? You bet. Here would be the ordering: 5—K Q; 4—J P; 3—
M N; 2—L; 1—O. Since answer choice (A) can be true, it’s the right answer; as quickly as
that.

(B) and (D) As noted above, in this question J and P are the only residents on the fourth
floor.

© K A PL A N 53
LSAT PREP ________________________________________________________________ LSAT Test II Explained: Section III

(C) Nope, N must live on either the first or third floor.

(E) Nope, P must live on the fourth floor.

• Don’t just stare at the new concrete information that a question stem provides: Jot it
down. A quick sketch inserting J on floor 4 and K on floor 5, here in Q. 8, pays off in
a major way. (And takes hardly any time at all to create!) Rethinking is a waste of time,
not resketching.

• It’s to be hoped that, on test day, you wouldn’t have bothered checking the other
choices after finding that choice (A) is possible. Learn to trust your work. Otherwise,
you’ll have a hard time making it to all 24 questions, or even the lion’s share of them,
in the allotted time.

9. (E)
O lives alone on the second floor. Jot that down, but we can’t deduce much from this, so
we’ll probably have to check each choice. Since we’re looking for the choice that cannot be
true—that is impossible—we can eliminate a choice by finding an instance in which its
given conditions work.

(A) Can K live on the fourth floor with O on the second? Sure, here’s the ordering: 5—M N;
4—J K; 3—P Q; 2—O; 1—L.

(B) Can K live on the fifth floor with O on the second? Sure: 5—J K; 4—P Q; 3—M N; 2—O;
1—L.

(C) Can L live on the first floor with O on the second? Sure; check the ordering under
either (A) or (B).

(D) Can L live on the third floor with O on the second? Sure, here’s the ordering: 5—J K;
4—P Q; 3—L; 2—O; 1—M N.

(E) is all that’s left and must therefore be the answer. For the record, why can’t L live on the
fourth floor with O on the second? Because if L is alone on the fourth (L is always alone no
matter where she lives) and O is alone on the second, we no longer have two consecutive
floors for K and P to occupy (Rule 2). (E) is the answer.

• If you spend time plotting out the scenarios contained in the choices, look to save
time by using the first few arrangements to eliminate as many choices as possible.
Notice how the arrangements in both (A) and (B) are sufficient to help us eliminate
answer choice (C).

54 © K A PL A N
LSAT PREP ________________________________________________________________ LSAT Test II Explained: Section III

10. (C)
Since M lives on the fourth floor, so must N. What next? Look at K and P, the other “bloc”
we know about. With the fourth floor occupied to the max, K and P must take floors 3/2, or
2/1. Either way, one of the K/P pair will occupy the second floor—scan the choices—
meaning that L, who lives alone on a floor, cannot live on the second floor as (C) suggests.

• When you’re stuck, ask yourself: Which entities do I know the most about? Which
ones work as a solid “bloc?” The answers you come up with may jump-start your
solution.

11. (B)
As noted re: Q. 6 above, a “must be true” with no new information is usually testing a
deduction. However, these unusually complex answer choices signal that that’s not the
case here. Other than skipping Q. 11 altogether, there’s no alternative to checking each
choice, using your previous work to save time whenever possible.

(A) If J lives on the fourth floor, can Q live on the fifth? Sure, we saw that when we worked
out Q. 8: J on floor 4 and Q on floor 5. (A) is not a true statement.

(B) If O lives on the second floor, can L live on the fourth? No way, and we just worked
through this in Q. 9. With O alone on 2 and L alone up on 4, there’s no place for the K/P
bloc; this pair would have to be broken up (so that neither would infringe on O or L’s
space), and that’s forbidden. Answer choice (B) must be true and is thus the answer. For the
record, here’s what’s wrong with the remaining choices:

(C) In Q. 10, we placed N on the fourth floor along with M and saw that it was possible to
put K on the second. Thus we’ve already seen that (C) need not be true.

(D) We have not yet seen a situation in which K lives on the third floor with O on the fifth,
but we can contrive one: 5—O J; 4—M N; 3—K Q; 2—P; 1—L. (Of course, having already
spotted (B) as correct we need do no such contriving!)

(E) More use of previous work. In Q. 8 (A), P lived on the fourth floor and M lived on the
third. That proves that (E) here need not be true.

• Sometimes you’ll find you have no recourse but to resort to simply testing out the
choices. If this seems as if it will be too time consuming, fine; skip the question for
the time being, and move to easier, more concrete challenges (look at all that info in
the stem of Q. 12). However, if you work carefully and with no slipups, there should
be ample time to chart out the different possibilities, especially if you can use your
work on previous questions to save time.

• Interestingly, correct choice (B) is identical to the stem-plus-answer in Q. 9. Probably


the testmakers figured that no one would notice . . . or perhaps they were trying to
reward those who did.

© K A PL A N 55
LSAT PREP ________________________________________________________________ LSAT Test II Explained: Section III

12. (C)
So O lives on the fourth floor and P on the second, eh? Finally, the same kind of helpful
concrete information we got in Q. 8. With P living on floor 2 (alone, thanks to Rule 3), K
must live on floor 3 (Rule 2). The fifth and first floors are the only ones unoccupied, so M
and N must take one of them while L takes the other. That leaves Q and J to be placed, and
O and K (on floors 4 and 3, respectively) are in need of floor-mates; but remember Rule 5,
which says that O and Q must split up. O and J will have to share floor 4, leaving K and Q
on floor 3. (C) has it right.

(A) and (D) are possible only, while (B) and (E) are patently false.

• Don’t doubt your past work as you move forward in a game. Trust that, as you go
along, you’re becoming more and more accustomed to how the entities interact with
the rules. Rely on that familiarity; don’t start doubting it.

56 © K A PL A N
LSAT PREP ________________________________________________________________ LSAT Test II Explained: Section III

GAME 3 — Hannah and Her Cities


(Q. 13-17)

The Action: Don’t let the brevity of this game’s intro (one sentence!) fool you—as with all
games, it’s vital that you conduct a thorough overview, and there’s quite a bit packed into
that one little opening sentence. We need to consider Rule 1 as well to round out the
picture. Hannah spends a total of fourteen days in exactly six cities, each of which is
located in one of three countries. After reading the introductory sentence and rules, you
should have realized that you had a pretty strange game on your hands; all the more
reason to concentrate on nailing down the game’s action. Basically, we need to distribute
the six cities into the three countries and also to divide up the fourteen days between the
six cities—thus, an oddball kind of grouping game of distribution with a heavy numerical
element. Notice that the element we’re distributing has no name—the ”numbers issue” is
the star of the show in this one. In most games we ask who goes where; in this one, how
many go where will be the operative question throughout the game. It may seem a little
convoluted, but games are always easier once we pare them down to their Key Issues, and
here they are:

1) How many cities are visited in each country?


2) How many days are spent in each city, and by extension, in each country?

The Initial Setup: The main challenge here is the number element, but we do need to
decide how to keep track of it all. At the start, we know that there are fourteen days spent
in six cities, and (Rule 1) that the six cities are distributed among three countries. You can
draw six dashes across the page and jot down a reminder of the total days, but that’s hardly
progress. Best advice: Don’t draw anything yet. Wait and see where the rules lead.

The Rules:

1) Again, those six cities that Hannah visits are each in one of three countries, X, Y, or Z.
Since we are going to distribute cities among those countries, set up a column or list for
each.

2) and 5) are loophole closers. Each country can have any number of cities—no limit on
cities per country. And “a day in a city” means just that: Hannah will not spend the
morning in a country X city, for instance, and the afternoon in a country Y city.

3) Each country must be assigned at least one of the six cities. Draw a single dash under
each country so as to remember the minimum per country. Make a mental note, too: Three
cities are accounted for; the other three remain.

© K A PL A N 57
LSAT PREP ________________________________________________________________ LSAT Test II Explained: Section III

4) If Hannah spends at least two days in each of the six cities she visits, then 2 x 6 = 12 days.
Of the 14 days, then, 12 are accounted for: a minimum of two days per city. What about the
remaining two days—can they be spent in a seventh city? No—we cannot assign Hannah a
seventh city; make sure you remember that she visits “a total of six cities.”

Key Deductions: You probably embarked on Step 4 of the Kaplan Method—combining


the rules—even as you read Rule 4. With twelve of the fourteen days already spoken for (6
cities x 2 days per city), Hannah has two remaining days that she can spend in any of the six
cities. What are the possibilities? There are only two. Either:

• She will spend 4 days in one city, and two days each in the other five: (4 x 1) + (2 x 5) =
14; or

• She will spend 3 days in each of two cities, and two days each in the remaining four: (3
x 2) + (2 x 4) = 14.

That’s it! These are the only two possible ways to distribute the fourteen days.

The Final Visualization: Here’s the information condensed into one sketch:

14 days total

X Y Z

2 2 2 2 2 2
+ 2 extra days

The Big Picture:

• This game beautifully illustrates why you can’t depend on sheer drawing for your
Logic Games success. Understanding the action and working out the arithmetic is
far more important than any picture you could create.

• While this section begins with a pretty straightforward game, other LG sections can
lead off with a “killer.” Try to locate the tougher games and save them for the end of
the section, after you’ve accumulated some much needed points and extra time and
confidence on the other games.

58 © K A PL A N
LSAT PREP ________________________________________________________________ LSAT Test II Explained: Section III

• Not all rules come in indented form. The introductory “paragraph” is just one
sentence, but it still contains vital information. Hannah spends her fourteen days in
exactly six cities. If you had glossed over these facts, you would have been sunk
before you started.

• The very fact that the game mentions an unusual number (at least for Logic Games)
like “fourteen” should be taken as a major hint that the game is going to hinge on
arithmetic. And when games focus so heavily on numbers, there’s usually a limited
number of ways that the numbers can play out—it’s worth your while to investigate
these options in advance.

The Questions:

13. (A)
There are only a few ways for Hannah to spend exactly eight days in Country X: We can
add the three extra cities from the Final Visualization above to the city already listed under
X, making a total of 4 cities in X, two days spent in each, for a total of eight days in X.
Keeping one city in Y and Z adds up to the requisite six cities, and the extra two days can
then be added to either Y’s city or Z’s city to account for the full 14 days. But it’s the other
possible distributions that eliminate all of the wrong choices: The following possibility —

X—4/2/2
Y—2/2
Z—2

— kills choices (B) and (D) by showing that Hannah can visit two cities in Y, and more cities
in Y (2) than Z (1). If we simply switch the situation in Y and Z —

X—4/2/2
Y—2
Z—2/2

—we see that both (C) and (E) could be true as well. The only choice that CANNOT be true
is (A): Hannah must visit exactly three or four cities in country X, not two, if she wishes to
spend exactly eight days there.

There’s another way to get this point: Maybe you simply went directly to the choices and
found (A) to be impossible: If Hannah visits exactly two cities in X, there are three choices:
Two cities for two days each, two cities for three days each, or one city for four days and
one for two:

X—2/2 or 3/3 or 4/2

No matter what, the most Hannah gets is six days in X, not eight as called for in the stem,
confirming that (A) cannot be true under these circumstances.

© K A PL A N 59
LSAT PREP ________________________________________________________________ LSAT Test II Explained: Section III

• Exactly eight days in X . . . The first question you must ask yourself is: “How can this
happen?” Don’t worry if you have to spend a little extra time working out the
possibilities—you’re still getting acclimated to the game and its unusual action. More
than just getting you the point, the time spent here will also go a long ways towards
solidifying your conception of the game. This, of course, will pay off in the
questions to come.

• If you’re at a loss, there’s nothing wrong with a little trial and error. Those who went
this route here were lucky; the answer came right away in choice (A). However, this
won’t always be the case. If the answer is buried down in choice (D) or choice (E), it
may take a while to get there. In any case, the proactive method is best; when given
new information, look to see how far you can take it before moving on to the choices.
Although you’ll come across some exceptions, in the long run, this strategy is your
best bet.

14. (D)
“An equal number of cities in each of the countries” is just the testmakers being coy: There
are six cities to be visited in the three countries, so “an equal number” means exactly two
cities per country. At least two days are spent in each city, so that makes four days
minimum spent in each country. 4 x 3 = 12, meaning that at this point two of the 14 days
have yet to be assigned, and there’s no reason why both can’t be spent in country X. Hannah
can spend up to six days in country X, choice (D).

• Make abstract information concrete. For example, translate a phrase like “an equal
number of cities in each of the countries” into something helpful you can work with.
Two cities per country is the correct interpretation, and is much more valuable when
it comes to getting this point.

15. (D)
Take this question stem one step at a time. Hannah spends exactly three days in country Y.
Well, she has to spend at least two days in each city, so Hannah can visit only one city in
country Y. (Make sense? Two cities would require at least four days.) Jot that down: 3 days,
1 city in Y.

Next: She spends seven days in country Z. Given that 2-day minimum for any city visit,
Hannah must visit exactly three cities in Z—one for 3 days, the other two for 2 apiece. (3 + 2
+ 2 = 7.) No other way to distribute the 7 days of country Z. At this point the right answer is
gettable, but let’s assume you didn’t stop to notice that (few would).

We’re left with four days to be spent in country X, and two cities left to be accounted for
(remember, we know there’s 1 city in Y and 3 in Z). Clearly she spends two days in each of
country X’s two cities.

60 © K A PL A N
LSAT PREP ________________________________________________________________ LSAT Test II Explained: Section III

Scan the choices against these deductions. Hannah can’t visit exactly two cities in country
Z—it has to be three. (D) must be false and is the answer. All of the other choices are true
statements.

• Drive ALL information to completion; even some innocuous-sounding statements


often mean more than they seem to. Here, “three days in Y” literally translates into
“one city in Y,” as discussed above; there’s simply no other way. Similarly, “seven
days in Z” in these circumstances means “three cities in Z,” which leads right to the
answer.

16. (B)
One of the six cities now has a name: Nomo. Hannah wants to spend as many days as
possible in Nomo, which is in country X. Under Key Deductions, we saw that since Hannah
must spend at least two days in every city she visits, the most days she can spend in any
one city is four. So we can infer that she spends four days in Nomo. Make Nomo the city
already in X from our master sketch, and add the two extra days to give Nomo a total of 4.
This means she’ll have to spend two days in each of the other five cities, to get to the total of
14. We were told that Nomo is in country X, so as long as Y and Z are assigned at least one
city each, the other three remaining cities yet to be placed in our master sketch can go in of
any of the three countries. Keeping all of this in mind, check the choices.

(A) Nomo needn’t be the only city in country X. In fact, fully four of the six cities could be
in X, leaving one each for Y and Z.

(B) Be careful of the wording here. If (B) is a true statement, then Hannah can visit four
cities in country Y. And she can! Say Hannah visits only Nomo in country X, and exactly
one city in country Z. Under those circumstances, she certainly could visit four cities in
country Y (just add the three floater cities). So (B) must be true and is the answer. Here is
why the remaining choices needn’t be true:

(C) Hannah will spend only four days in Nomo. As we’ve seen, four days is the maximum
for any city, so six is impossible.

(D) Sure she can; actually, Hannah could spend up to eight days in country Z (4 days in X’s
Nomo; 2 days in a single Y city; and 8 days spread throughout 4 Z cities).

(E) Actually, Hannah can visit at most five cities in Y and Z combined—as long as she
spends the required four days in X’s Nomo.

• Stay on your toes and continue to use common sense. This question employs a form
or wording we’re not familiar with but still isn’t difficult to understand. If I say “It
MUST be true that Tom COULD be the thief,” what I’m really saying is that “it
COULD be true that Tom is the thief.” Likewise with (B): If the question asked
“which of the following could be true?”, then the choices would have been worded
normally and (B) would read “Hannah visits four cities in country Y.” All that’s
required is a little translation, and that of course hinges on the crucial skills of
critical reading and critical thinking.

© K A PL A N 61
LSAT PREP ________________________________________________________________ LSAT Test II Explained: Section III

17. (C)
If Hannah visits four cities in countries X and Y, then she must visit two cities in country Z
(all 6 cities are now accounted for). She spends at least two days per city, so she spends at
least four days in country Z. That leaves at most 10 days that she can spend in countries X
and Y, and of these ten days, Hannah must spend at least two days in one city in country X.
This leaves a maximum of eight days that she can spend in country Y, choice (C).

• The numbers aspect of Logic Games is always important. This game, however, was
all about the numbers aspect. You had to keep in mind that fourteen days were spent
in six cities, and that at least two days were spent in each city. If you remembered all
of this, the game was very doable. When a game hinges on arithmetic, don’t cut
corners. Work it out.

62 © K A PL A N
LSAT PREP ________________________________________________________________ LSAT Test II Explained: Section III

GAME 4 — The Dog Show


(Q. 18-24)

The Action: After reading the introductory paragraph and rules during your game
overview, you probably thought, “Whew! There’s a lot going on here.” And so there is. Six
dogs are each either a greyhound or a labrador and each either male or female—the
matching aspect of this complex hybrid game. The six dogs are awarded four ribbons, first,
second, third, and fourth based on their performances in the dog show, while two of the
dogs don’t get a ribbon at all. The ribbons make up the sequencing part of this hybrid
game. The Key Issues are:

1) What is each dog’s gender and breed?


2) What ribbon, if any, does each dog receive?

The Initial Setup: One of the big challenges inherent in this game is how to keep track of
all the information. The ultimate decision about this will come when you deal with the
rules. For now, list the dogs and the possibilities for each dog’s gender, type, and ribbon.
Note: it’s a good idea to include “Xs” to represent the two dogs who won’t get a ribbon:

P Q R S T U 1
2
Grey Lab 3
f m 4___
X X

Now we’re going to go through the rules step by step, just as you might have done when
working on the game yourself. If you had trouble with this game, please take your time and
follow this discussion carefully. The effort will be worth it. We promise.

The Rules:

1) simply sets the possible type of each dog. Each of the six dogs is either all greyhound or
all labrador; no mixed breeds are possible. We’re not told how many of each breed are
present, so there’s not much more to do with this rule right now.

2) Here’s a helpful numbers rule. Exactly two of the six dogs are female, and exactly four
are male. Make a note of that: You need to distribute two f’s and four m’s. (Or you can use
the universal male/female symbols for that purpose.)

© K A PL A N 63
LSAT PREP ________________________________________________________________ LSAT Test II Explained: Section III

3) This rule has a lot of information, so as always with two-part rules, break it up and take
it one step at a time.

First of all, both female dogs—there are only two females, remember—are awarded ribbons.
It’s a very small step to realize that both of the dogs who don’t receive ribbons must
therefore be male. (Make a note of that: Those “X” dogs, the losers, are male.) By the same
token, the other two ribbon winners must be male too. So make a note next to the 1st
through 4th place slots: Two are female, two are male.

Secondly, we’re told that exactly one of the two female dogs is a labrador. Again, it’s not a
big jump to recognize that the other female dog must be a greyhound. To sum up (always
a good idea in a complex game): We have two females finishing in the top four and
winning prizes; one is a greyhound, the other a lab. We also have two winning males, and
two loser males, breeds as yet undetermined.

4) states that exactly one labrador receives a ribbon. This should sound very familiar,
because just one rule ago we saw a labrador winning a ribbon; it was a female. Well, Rules
3 and 4 must be talking about the same ribbon-winning labrador. So since we have a single
winning labrador, therefore the other three winners must be greyhounds. Our winners, then,
are two male greyhounds, one female greyhound, and the selfsame female labrador. Those
are the dogs that will take 1st through 4th place, though to be sure we don’t yet know their
“names.” (It’s worth noting another implication of this rule: Since there is one and only one
winning labrador, and it’s a female, any male labrador that happens to wander by in a
question will have to take one of the two “loser” slots.)

5) This is the rule that most explicitly deals with the sequencing aspect of this game. It’s
also the rule around which the master sketch will truly be built. Sketch it out rather than
trying to work with it on a sheer mental level: Since P and R place higher than S who places
higher than Q and T, draw it that way. Create a P and R with arrows down to an S, then
more arrows down from that S to a Q and T.

The big question that ought to come to mind, and bravo to you if you thought to ask it, is:
What about U, the only dog unnamed by Rule 5—where does U place? The answer, for the time
being, is: Don’t know. But we will.

6) P and R are greyhounds. Fine; let’s just jot that down for the moment. Next to the P and
R that you’ve just drawn, note each as a “grey” to remind yourself of these dogs’ types.

7) Same kind of thing: S and U are labradors. Next to the S in your master sketch add a
“lab” for labrador. As we noted before, we haven’t placed “U” in the master sketch yet, so
jot down “U=lab” or something; but as they say, hold that thought . . .

64 © K A PL A N
LSAT PREP ________________________________________________________________ LSAT Test II Explained: Section III

Key Deductions: Now it’s time to put it all together, and as usual, we should look at our
most significant “bloc” of information. Here, it’s got to be the sequencing information from
Rule 5. Use it to pursue the question we asked a while ago regarding Rule 4: What are the
winners’ names?

Can P and R not win ribbons? No way. Each of them places higher than three other dogs,
according to Rule 5. So P and R win ribbons.

Can S not win a ribbon? No way. S places higher than Q and T, so at the very worst, S can
finish no lower than 4th place. Now we have three winners named out of four: P, R, and
S.

Who’s the fourth winner? Well, it could be either Q or T; whichever one wins a ribbon, the
other is a “loser.” But what about U? Can U be the fourth winner?

No way—and that’s the whole ball of wax, the Big Deduction available here. Remember
what Rule 3 told us (and your sketch should remind you of it, if nothing else): There’s only
one ribbon-winning labrador. S, a labrador, is that winner—we just deduced that from
Rule 5. Therefore labrador U must take one of the “loser” slots. Go back over these last few
steps carefully, to make sure you understand ‘em. Let’s sum it all up:

• Our winners are P, R, S, and one of the Q/T pair. U, and either Q or T, are the losers.
Who gets which ribbon? Read on:

• Given the demands of Rule 5, P and R must take the 1st and 2nd place ribbons, in
either order. S will have to take 3rd place, leaving either Q or T to take the 4th place
ribbon. Once again, of Q and T, the one who doesn’t win shares “loser” status with U.

• We also know that S, as the winning labrador, must be female. (Rules 3, 4, and 5
demand it.) Thus the other winning female, a greyhound, is either P, R, or the 4th place
dog. That’s another bit of uncertainty. However, we do know that since there’s exactly
one winning labrador (Rule 4), and that S is it, then the other ribbon winners must be
greyhounds. No big deal for P and R—we already knew they were greyhounds from
Rule 6. But this does allow us to deduce that the fourth place dog, be it Q or T, must
be a greyhound.

• What about the losers’ breeds? One loser, U, is a labrador, we know that. The other
loser, male, could be labrador or greyhound—no way to tell.

• Both “loser” dogs, of course, are male—we deduced that way back at Rule 3, but it’s
been a while since then. . . .

© K A PL A N 65
LSAT PREP ________________________________________________________________ LSAT Test II Explained: Section III

The Final Visualization: Here’s your master visualization of the whole thing:

1/2 1/2
P Grey R Grey
Grey
Lab 3
f S Lab
ffm m mm
4
Grey
1 2 3 4 x x Q T
X
m U Lab

The Big Picture:

• “How could I possibly get all of that?” some students wail. Well, it’s a good question.
The answer is: Take things step by step; don’t just draw, but think about the
implications of rules; keep remembering what the game’s action is and work to
achieve it; jot down what you learn in a helpful way and rely on it to propel you
forward; and above all, don’t doubt your work midway. Keep moving forward.

• A second complaint: “How can anyone possibly get it done in time?” An even better
question. Clearly this game demands a lot of time in the setting up. But if you have
used your time on the other three games wisely, you should have more than the
average 8 minutes to work this out carefully. And if you do work it out carefully, if
you commit to creating a solid foundation, you’ll find that most of the questions fall
quickly and easily.

• An element of trust is required here—trust that (1) the game wouldn’t be on the
LSAT if it weren’t somehow doable, and (2) if you hang in there and work things out,
some if not all of the questions ought to be readily available to you.

• A good Kaplan rule-of-thumb: Save single letters for the entities or easily intuitive
distinctions (like “m or f” for male or female), and spell everything else out for
yourself. The extra nanoseconds you spend writing “grey” and “lab” instead of “g”
and “l” can pay off in a lot more confidence and a lot less confusion.

66 © K A PL A N
LSAT PREP ________________________________________________________________ LSAT Test II Explained: Section III

The Questions:

18. (E)
Which dogs can be greyhounds? Well, why not start with the dogs that we definitely know
must be labradors? S and U are the only dogs that are explicitly identified (by Rule 7) as
labradors, so any of the other dogs can be greyhounds. That’s P, Q, R, and T, answer choice
(E). (Of course P and R are greyhounds for sure, but they still merit a place in the right
answer.)

• It’s not unusual to be rewarded with a simple point just for making it through the
setup. We made a ton of deductions, but this point is ours for the taking simply by
applying Rule 7.

19. (B)
Not much to do with this “CANNOT be true” question but to try out the choices.
Remember that since the right answer is impossible, the four wrong choices all could (or
must) be true.

(A) Either P or R (both greyhounds) wins 2nd place, and either can be female. (A) need not
be false.

(B) This is it. Only P and R may win the top two ribbons, and Rule 6 identified P and R as
greyhounds, so there’s no way that a labrador of either gender could win the second place
ribbon. Since (B) can’t be true, it’s the answer. For the record, here’s the skinny on the
remaining choices:

(C) S, a female lab, must win the third place ribbon.

(D) and (E) Either Q or T could be a male greyhound winning 4th place. By the same
token, either could be female. Remember, we still have one female ribbon winner
unaccounted for.

• Even when a question requires you to try each choice, quality work up front greatly
reduces the time required to eliminate each wrong answer.

• Another timesaver: Characterize the right and wrong answers before you jump in.
Mentally confirm what you know about the one right and four wrong choices; doing
so will really save you time and points.

© K A PL A N 67
LSAT PREP ________________________________________________________________ LSAT Test II Explained: Section III

20. (E)
This “must be true” question with no new information is testing for one of our deductions.
We deduced many things, but the only dog that we deduced must be male is U, choice (E).

• Even though this question came third, you might have tackled it early on had you
seen it. A big reward for a Big Deduction.

• By the same token, had you gotten stuck during the setup work, the very wording of
this question could have been a stop sign telling you to go back and look closer for
more deductions. After all, if they can ask in Q. 20 “which dog has to be male,” that
has to mean that there’s more gender identification work to be done—there must be
at least one Big Deduction there for the plucking.

21. (A)
You’re looking for the choice that can be false, so the four wrong choices must be true.
There’s no new information, so not much to do but to hit the choices.

(A) P and R win the first and second ribbons, but P doesn’t necessarily have to come in
ahead of R. R could win first and P second. Since (A) could be false, that’s all she wrote; the
remaining choices all must be true as the master sketch clearly shows.

• During test day’s Logic Games, don’t bother checking the remaining choices once
you’re satisfied with your answer. You can’t afford to waste time, and you don’t get
any extra credit for simply confirming incorrect answers. (The same advice does not,
of course, always hold for Reading Comp. and Logical Reasoning.)

22. (E)
If Q is female, then Rule 3 says that Q must receive a ribbon. S is the one labrador that wins
a ribbon, so, thanks to Rule 4, Q must be a greyhound. There are only two females, S and Q,
so the remaining dogs, P, R, T, and U, must all be male. From here, it’s a simple matter of
scanning the choices and looking for the one choice that doesn’t match up, and it’s (E): T, a
losing male in this case, could be a greyhound, but he could also be a lab. (E) could be
false and is the answer.

• This is the game’s fifth question, but the first question with an “if” clause in the
stem. This means that the first four questions could be answered solely from your
up front work, which is another big argument for taking the time to properly do the
initial work.

68 © K A PL A N
LSAT PREP ________________________________________________________________ LSAT Test II Explained: Section III

23. (B)
This time it’s T who wins the fourth place ribbon. Rule 4 is still in effect, so T must be a
greyhound. Q and U are the two dogs who don’t win ribbons, and we deduced from Rule 3
that any dog who doesn’t win a ribbon must be male. Q must be male, choice (B).

(A) Not necessarily true: P could be the second ribbon-winning female.

(C) Ditto. T could be the second ribbon-winning female.

(D) Not necessarily true. As a loser, Q could just as easily be a greyhound.

(E) False. We’ve already deduced that T is a greyhound in this question.

• After you’ve deduced something new, take a break and quickly scan the choices
looking for your new deduction. You may save valuable time that way.

24. (D)
Another question requiring a browse through the choices, once you’ve worked out that
since the right answer could be true, the four wrong choices must be false. Actually, (A), (B),
and (C) have been worked out long ago. Rule 5 made it clear that P, R, and S are definite
ribbon winners, so none of those statements can be true; and since we deduced some time
ago that U is a ribbon loser, (E) is impossible as well. (D) is left, and of course it’s possible:
T might win a ribbon, or might not.

• Another “non-if” question answerable solely from your work on the setup. That’s 5
“non-if’s” out of this game’s 7 questions, proving once again that any time spent
making careful deductions that lead to right answers couldn’t be better spent!

• The testmakers reward clever test-takers who notice even the little details. Did you
notice that the answer to this question can be found right in the question stem of Q.
23? “If dog T wins the fourth place ribbon . . .” certainly means that it’s possible for T
to win a ribbon. In a “could be true” question, scanning the choices for something
that looks familiar (either from a question stem or your previous work) may lead to a
quick and easy point.

© K A PL A N 69
SECTION IV:
LOGICAL REASONING

70 © K A PL A N
LSAT PREP ________________________________________________________________ LSAT Test II Explained: Section IV

1. (C)
On what does the author base his conclusion that the theft was carried out in order to add
to the collection of a private collector? The only evidence available to the author is the
selection of art works on the part of the thieves. The author must be relying on the principle
described in (C): from the pattern of works taken (in this case, the works had been
“carefully selected,” but not on the basis of monetary worth), it is sometimes possible to
distinguish one type of art theft from another (in this case, to perceive that a theft was
carried out in order to please an individual collector).

(A) The author inferred that this theft was carried out to suit a private collector. He needn’t
assume that whenever an art theft takes place (including thefts where only the most
expensive pieces are stolen), he can tell whether one or many “known” individuals
“directed” that theft.

(B) The author never claimed that the pattern evidenced by this theft “defied rational
analysis;” in fact, since a very definite plan was evidently carried out, the opposite seems
to be the case.

(D) Suppose, contrary to (D), that thefts without a preexisting plan for the disposition of
the stolen works did uniformly involve the theft of only the most valuable pieces: that
wouldn’t at all hurt the author’s argument that the failure to select the most valuable works
in the case of this theft shows that there is a preexisting plan to get the works to a private
collector. So (D) isn’t appealed to.

(E) The author never even mentions the “integrity” of the remaining collection, so he
needn’t appeal to (E).

• The question asks, in effect, for a principle that is assumed by the argument. You can
use the Denial Test on the choices, exactly as you do on assumption questions. Deny
the “principle”; if the argument still holds, then the principle isn’t assumed or, as
they put it here, is not “tacitly appealed to” by the argument. (Conversely, deny
correct choice (C) and the argument falls apart.)

• Recognize the limitations of the author’s argument—he’s just saying that, in this
particular unusual case, he can infer something about the theft. Avoid choices, like
(A) and (B), that use absolute terms like “any” or “every.”

2. (C)
The last band on all the pigs’ teeth was translucent. That means these last bands were
deposited in the winter. But these bands are only half the normal width; since the
depositing of cementum is supposed to be “constant,” this means that the bands were
deposited in only half the normal time. In other words, the pigs stopped depositing
cementum about half-way through the winter. According to what we’ve been told, we
would expect a pig to continue depositing cementum at a constant rate as long as it’s alive,
so we can reasonably infer (C), that the pigs died about half-way through the winter.

© K A PL A N 71
LSAT PREP ________________________________________________________________ LSAT Test II Explained: Section IV

(A) conflicts with the evidence. If the pigs had died in an unusually early winter, then the
previous summer would have to have been unusually short. If the previous summer had
been unusually short, then the opaque band deposited at that time would have to have
been unusually narrow. Since all the bands other than the last were of constant width, the
previous summer was not unusually short. In turn, this implies that the fatal winter was
not unusually early.

(B) We just don’t know the ages of the pigs at the time of their death; we could only
calculate their ages if we’d been told the total number of rings in their teeth. As it is, we
have no idea.

(D) misinterprets the evidence to suggest that all the pigs died at the exact same time,
rather than that they all died at about the same time of year. In other words, (D) seems to
suggest that all the pigs bought the farm mid-way through the same winter, but this need
not be the case.

(E) goes beyond the evidence. Starvation is one of the things that could have killed the pigs
in the winter, but we don’t know that it did; maybe they froze to death or they’re more
vulnerable to predators in the snow.

• One thing that’s worth remembering about LSAT Logical Reasoning questions
based on scientific stimuli is that they’re all pretty reasonable. Unless they’re giving
you a speaker’s opinion or a fictional scientist’s theory, they’re not going to ask you
to infer something that’s flatly contradictory to nature. Here, for example, it’s
perfectly reasonable that pigs tend to expire in the middle of winter, which is
generally the most difficult season for animals.

3. (E)
Reading the question stem tells us that we have an imposing task: we need to identify the
one thing out of five that the author doesn’t do. The best way to handle this kind of question
is to attack the stimulus looking for the methods the author uses to prove her point. You
can then eliminate the answer choices that contain methods you recognize. So what doesn’t
the author do? The author’s entire argument is based on description; the author tells us
what happened and how it has affected the current situation. However, the author never
addresses the ethics of the situation. In particular, the author does not, as answer choice (E)
suggests, question the moral foundation of the economic conditions she describes.

(A) The author does draw an analogy when she compares the notion of a country living
without foreign trade to a dog living by eating its own tail.

(B) The author notes that the United States has found most of its raw materials and
customers for finished products within its own borders, so she does appeal to a historical
fact.

72 © K A PL A N
LSAT PREP ________________________________________________________________ LSAT Test II Explained: Section IV

(C) and (D) The author claims that the United States’ practice of looking only within its
own borders has led to the current situation: a large foreign debt and the United States’
current status as a foreign playground.

• Asking “what is true” is different from asking “what should be true.” Often an
author will present a description of a situation but will not address the moral issues
connected with the situation. In some cases, wrong answer choices will claim that an
author does have an opinion on ethics when he or she really doesn’t; you should
eliminate these with confidence. In an all EXCEPT question, this choice becomes
your answer.

4. (A)
Giselle’s goal in increasing the sales tax on gas is less consumption of gas. She wants to make
gas more expensive so that people use less of it, and she suggests raising the tax on gas as a
way of doing so. Antoine misses her point entirely; he complains that it’s not fair to make
gas users bear the entire burden of a tax increase, which should instead be spread among
all the people. But Giselle’s whole reason for raising the gas tax is precisely in order to stick
it to gas users so they would cut consumption. Antoine’s reply ignores Giselle’s stated
reason for raising the gas tax; as (A) says, he ignores the fact that Giselle is not simply
interested in raising more revenue for the government but has a non-revenue related
motive.

(B) is unimportant. Antoine’s mistake is bringing up non-gas users in the first place, not
failing to specify how many there are.

(C) The problem isn’t that “unfairness” is an inappropriate criterion when considering a
tax hike. The problem is that Antoine calls Giselle’s plan “unfair” because he has no idea
what she’s trying to accomplish; she’s not trying to raise revenue, as he thinks, she’s trying
to cut gas consumption.

(D) On the contrary, Giselle does indeed only want to raise the sales tax on gasoline, and
Antoine’s problem is that he doesn’t understand why.

(E) is off on two counts: first, in calling it “implausible” that there might be a better way to
spread the burden of raising taxes, and second, supposing that it matters to Giselle’s
argument whether the burden is spread evenly or not.

• The chief difficulty in this question comes from the wording of the answer. You
probably saw that Antoine missed the fact that Giselle based her argument for
raising the gas tax on a need to lower gas consumption. (A) doesn’t state this
positively, but in a roundabout manner, saying that Antoine missed the fact that
Giselle did not base her argument on a need for increased revenue.

© K A PL A N 73
LSAT PREP ________________________________________________________________ LSAT Test II Explained: Section IV

5. (B)
Another airline question—back in Section 2, airplanes were crashing, but now they’re
merely late. This question asks us which choice, if true, would challenge the validity of the
agency’s rating system. In challenging the validity of the ratings, you need to keep in mind
what the ratings are intended to do. This is another way of saying that you need to keep
your focus on the scope of the argument. The ratings are designed to measure the airlines’
relative efficiency, and they do this by calculating each airline’s number of late flights as
compared to their total number of flights. Does this sound like it would be a fair measure
of efficiency? Maybe not, since some factors that cause late flights are not under the
airlines’ control. If one of those factors, like bad weather (choice (B)), affected some airlines
more than others, then the rating system wouldn’t be so fair after all.

(A) is irrelevant. If travelers have no choice, then the rating system may not help them, but
that is a different issue from the question of whether the ratings measure what they are
intended to measure.

(C) and (E) cite scheduling details that apply to all airlines but don’t give us any reason to
suspect that some airlines would be unfairly slighted.

(D) is also irrelevant. It is hard to believe that making the airlines aware of the monitoring
would cause the results to be skewed in some way, especially since the airlines
presumably have some incentive to do a good job already.

• Always focus on the scope of the author’s argument. Arguments can be


strengthened or weakened on their own terms, and no others. An answer choice that
makes a point related to a different argument is irrelevant, no matter how interesting
it is.

• It pays to read the stem first. In this case, we learn not just that we have to challenge
an argument, we’re also alerted in advance that a ratings system is in place for a
particular purpose.

6. (B)
Welcome to “Jumping to Conclusions 101.” Our lesson today concerns what not to do
when you have two plausible explanations for the same event. We know that when you mix
acidic liquids with baking soda, you get fizzing. We also know that the contents of a bottle
labeled “vinegar” produced no fizzing when mixed with the contents of a box labeled
“baking soda.” In that case, according to the author, it must be that the bottle has been
mislabeled. Is that fair? Based on the information in the passage, it is equally likely that the
box might not have contained baking soda. The author completely ignores this alternative
explanation and so commits the flaw in (B).

(A) is irrelevant. Even if the bottle contained another acidic liquid, there would still be no
fizz, and we’d be back where we started: the experiment as described still would not be
able to show that the bottle was mislabeled, since the box might have been mislabeled.

74 © K A PL A N
LSAT PREP ________________________________________________________________ LSAT Test II Explained: Section IV

(C) A fizz is a fizz is a fizz—there’s not much one can do with this term. In other words,
there’s nothing imprecise here about the author’s use of the term “fizz”; it’s used the same
way throughout the argument.

(D) The scientific principle (if you combine certain elements, the result will fizz) is not in
question here, so a consideration of proper “laboratory conditions” is unwarranted. The
author commits a logical error; how to best test “scientific principles” is beyond the scope.

(E) is irrelevant. The intent to deceive is never an issue here. We need to know if the bottle
is mislabeled, not why.

• Alternative explanations are the key to many questions, including Assumption,


Strengthening and Weakening questions. In this case, ruling out a plausible (and
fairly obvious) alternative explanation is a flaw.

• Choices that claim that an argument is flawed because it uses an ambiguous term
(like “fizz” in this case) are almost always wrong.

7. (D)
This is another question that takes some twists and turns, so you’ll need to paraphrase the
argument to keep on the path. The question stem tells us to look for evidence that would
support the new theory over the old, so we have to be careful to keep them straight. The
old theory claims that shell color evolved as a means of disguising snails from predators;
snail shells would match the color of their surroundings, leading to a camouflage effect.
The new theory holds that shell color is related to heat absorption. It predicts that brown
shells will be more common when wave conditions make heat absorption more difficult,
with light shells being more common in calmer waters. So, what kind of scenario would
favor the new over the old? It would have snail shells that did not match the color of the
surroundings but were related to wave action. (D), which posits light shelled snails in
dark, predator-filled surroundings, conflicts with the “predator theory.” It also includes
calm waters, which is in accord with the new theory’s assertion that light-shelled snails will
be found where wave action will provide less interference with the Sun’s rays.

(A) Au contraire, dark shells in a calm inlet with a dark bottom supports the “predator
theory.”

(B) and (C) don’t match the new theory’s prediction that light shells will be found in calm
areas and dark shells will be found in more turbulent waters.

(E) is consistent with either theory, so it can’t be a basis for favoring one over the other.

• Use your common sense when deciding on which questions to put off until later.
The stem is complex, and the passage is fairly long, and even the answer choices all
look the same and require a very careful analysis. You may have found it beneficial
to leave this one and return to it if time permitted at the end of the section.

© K A PL A N 75
LSAT PREP ________________________________________________________________ LSAT Test II Explained: Section IV

8. (C)
The amino acids in eggshells decompose more slowly in cold climates than in hot climates;
so much so in fact, that the technique of dating by amino-acid decomposition in eggshells
works for sites a million years old in cool climates, but only for sites up to 200,000 years
old in warm climates. Something may have occurred to you when reading this: “What if
the climate changed? Wouldn’t that make dating more difficult?” (C) makes this difficulty
even greater. What if the climate fluctuated a lot, and we didn’t even know about it? As (C)
says, that would inferably make the technique less likely to yield accurate results, since we
couldn’t allow for the varying rate of amino-acid decomposition.

(A) We’ve only been told about how far back the amino acid test will work in different
climates; we have no basis for inferring where the oldest sites are.

(B) is beyond the scope; the stimulus only talks about this one type of dating. For all we
know, not only does amino-acid decomposition take place in other organic matter, but it is
also used for dating archaeological sites.

(D) is way too specific. Not only do we have no idea what proportion of the amino acids in
an egg has decomposed at any given time, we don’t even know that when the acids
decompose they are “no longer suitable for examination by the technique.”

(E) is sheer fantasy; the stimulus discusses the rate of amino-acid decomposition in the
different climates, not the comparative likelihood of finding eggshells.

• The skills that work on one type of question can help you on other types of
question. Here it helps to be able to grasp the Main Point: amino-acid decomposition
is used to date old sites, and it varies according to climate.

9. (D)
Another standard assumption question here. Focus on the evidence and conclusion and
keep an eye out for what’s missing. The advertisement tries to convince us to buy Clark
parts and cites the fact that Clark parts, unlike foreign-made parts, have passed
government tests. So Clark parts are subject to government tests, whereas foreign-made
parts are not. So what? We still don’t know how tough those tests are. If parts that pass
those tests are just as flimsy as cheap foreign parts (the denial of (D)), then Clark parts don’t
look so hot after all.

(A) is irrelevant. The issue is the quality of Clark parts, not their availability.

(B) and (C) may strengthen the argument if true, but are not necessary for the argument to
stand. Even if some foreign-made parts were suitable for cars in the country (the denial of
(B)), or if some foreign-made parts did measure up to our standards (the denial of (C)),
Clark parts might still be the smart choice, and so neither (B) nor (C) is the necessary
assumption on which the argument depends.

76 © K A PL A N
LSAT PREP ________________________________________________________________ LSAT Test II Explained: Section IV

(E) guarantees that all parts made for cars manufactured in our country (including Clark
parts) are not poorly constructed. If some of those parts were poorly constructed (the
denial of answer choice (E)), then Clark still might be tops, since their parts need not be the
flimsy ones.

• Some assumptions may jump right out at you—if not, the Kaplan Denial Test
provides another way of testing the choices. In this question, it may be hard to see
why answer choice (D) is necessary until you think of what would happen if it were
false. Or, it may be hard to see why (B), (C), and (E) are wrong until you deny them
and see that the argument could still be valid.

10. (D)
Reading the stem first tells us that four of the choices will strengthen the argument and one
will not. Fortunately, the argument is not very complicated. The author concludes that
computer criminals are likely to avoid arrest and conviction even when their crimes are
detected and reported. The author provides no evidence to support this claim, and so
there’s nothing to do but tackle the choices. Choice (D) is out of the scope. The argument
deals with the prospects of arrest and conviction. What happens after conviction (which is
the only time sentencing occurs) is the topic of another argument altogether. All of the
other choices are directly related in some way to the unlikelihood of either arrest or
conviction.

(A) strengthens the argument by suggesting that prosecutors, responding to incentives to


get the greatest number of convictions, will not make prosecuting computer crimes a high
priority.

(B) If officers don’t have enough time to become proficient at computer crime
investigation, it is easier to believe that computer criminals will get off the hook.

(C), which states that computer crime is a relatively low priority for the police, makes it
less likely that the police will even investigate reports of computer crime.

(E) If incompetent officers inadvertently destroy evidence of computer crime, it stands to


reason that the perpetrators of those crimes would be less likely to be convicted.

• Always focus carefully on the scope of the argument. Here, the important terms are
“arrested” and “convicted.” Four choices deal with issues related to these terms. In
correct choice (D), however, the word “sentencing” should have jumped out at you
as being clearly outside the narrow confines defined by the two relevant terms in the
stimulus.

© K A PL A N 77
LSAT PREP ________________________________________________________________ LSAT Test II Explained: Section IV

11. (B)
What’s wrong with this picture? The movie review segment has drawn 10 negative letters,
and the news reporting/music selection has drawn 50 positive letters. From out of
nowhere, the programming director then makes the naked assumption that some people
did like the movie review segment. Huh? The fact that some people hated the segment does
not imply that some others did like it, and so the director commits the flaw in (B).

(A) suggests that the director has overreacted to the negative letters. He certainly doesn’t
have that problem.

(C) Since the favorable and unfavorable letters concerned different programs, a
comparison of the number of each is unwarranted. This is not the director’s flaw.

(D) What relation?

(E) It’s not clear why the director would have to wait until there were 50 negative
responses, when 50 was merely the number of positive responses to a different program.

• Don’t bend over backwards trying to make convoluted arguments for flawed answer
choices. Eliminate them with confidence.

• In Logical Flaw questions, don’t fault the author for not doing something he or she
is not required to do. In this question, wrong answer choices accurately point out
things that the director has not done, but since these things are not warranted in this
argument, none of those omissions amounts to a flaw.

12. (A)
Reading the stem first tells us tons of useful information: we know a doctor will provide
two reasons to do something that will seem inconsistent, and it will be our job to explain
how they could coexist. The doctor claims that medical staff will waste time retrieving and
returning files, and also claims that no one will ask for the files anyway. Can this
relationship be saved? Sure, we just need a choice that would explain why the need to
retrieve and return files would be independent of requests for those files. (A), which
suggests that files might have to be available whether they are requested or not, hits the
mark.

(B) and (D) are irrelevant. Both giving the job to the lowest paid worker and allowing
doctors to pass along the costs to consumers would make the job less expensive, but
neither explains how granting access will be time-wasting even if no one ask for the files.

(C) Suggesting that some file-seeking patients will take more time than originally thought
is still inconsistent with the claim that no patients will ask for the files, and so (C) doesn’t
resolve the problem.

78 © K A PL A N
LSAT PREP ________________________________________________________________ LSAT Test II Explained: Section IV

(E) supports the doctor’s claim that patients will not exercise their right to see their
records, which effectively works towards canceling out the first consideration. In other
words, (E), far from resolving the existence of the two considerations, only deepens the rift
between them.

• The question stem alerts you to the fact that two “reasons” may seem at odds, and
that it’s your job to find the choice that reconciles them. Knowing this up front
makes it easier to spot the potential inconsistency in the doctor’s argument, which is
the first step in locating the credited choice.

13. (B)
Hawthorne sold his influence with high government officials, says Alia; his behavior was
thus unethical and unjustifiable—end of story. On the other hand, Martha sees extenuating
circumstances: by selling his influence to benefit an anti-pollution group, Hawthorne
helped the public. Therefore, his behavior was not unethical. Alia believes Hawthorne’s
behavior was unethical in and of itself; Martha believes that the good consequences of
Hawthorne’s behavior ethically justifies it. (B) nicely describes this disagreement.

(A) Neither speaker makes any reference to the passage of time as having any effect on the
meaning of ethical behavior.

(C) Both speakers judge the ethical standing of Hawthorne’s behavior, so neither one
evidently believes that it’s inappropriate for others to impose standards for judging ethical
behavior on him.

(D) introduces a false distinction which doesn’t enter into the discussion. Both speakers are
judging the ethics of Hawthorne’s behavior in a public situation.

(E) Neither speaker discusses where the definition of ethical behavior is rooted; neither
says a word about religion or philosophy.

• It’s important to note when the author uses extreme language. In this question, Alia
claims that there can be no justification for Hawthorne’s behavior, and so Alia would
have to reject any potential explanation for that behavior. No ifs, ands or buts.

• Make sure you don’t read things into the argument. All of the wrong choices here are
clearly outside the scope.

• In point-at-issue questions, the right answer will always contain a concept or a


notion about which both speakers have a clear, and differing, opinion.

© K A PL A N 79
LSAT PREP ________________________________________________________________ LSAT Test II Explained: Section IV

14. (E)
Here’s another question in which milking the question stem really pays off. It tells us not
only that we should strengthen the argument, it also tells us exactly what that argument is.
OK, so what would show that changing the requirements for “top priority” cases was
responsible for the better response time? We know from the stimulus that the old
definition of “top priority” used to include the cases that took the longest time, so what’s
missing? We don’t know whether those time-consuming cases were a significant
proportion of last year’s “top priority” emergencies, but (E) fills in that gap. If the time-
consuming cases that are no longer part of “top priority” were fully half of last year’s “top
priority emergencies,” then we would expect better response times simply from the
redefinition alone.

(A) is too vague to be relevant. How long is the turnaround time for these emergencies?
What relation does this decrease have to the redefinition of the top priority category? We
don’t know these things, so this decrease by itself cannot strengthen the author’s
conclusion.

(B) is also irrelevant. The issue of financial priorities is unrelated to the evidence and
unrelated to the conclusion as well.

(C) is out of the scope. The question is whether the redefinition in fact made a difference,
and not whether “experts” thought it was a good idea.

(D) is also out of the scope. The issue is this city, not other ones.

• Pay close attention to the terms of the argument. Choices that change those terms are
irrelevant to the argument.

15. (A)
Another assumption question, with the conclusion clearly indicated by the Keywords “It
follows that . . . ” So the author claims that the pet lovers were voted down by a majority of
the tenants, but all we know is that their attempt to change the policy failed. They might
have been voted down by the majority, but they also could have failed to obtain the
signatures of the required ten percent of tenants that are required for a majority vote. Thus,
if choice (A) is false, then the argument is defeated, and so (A) must be a necessary
assumption.

(B) is the au contraire choice, since if the pet lovers had failed to obtain the required ten
percent, then a majority vote could not have taken place.

(C) is not required, since a majority of less that ninety percent would also be capable of
voting down the proposal.

80 © K A PL A N
LSAT PREP ________________________________________________________________ LSAT Test II Explained: Section IV

(D) weakens the argument. If the support of only ten percent guaranteed success, then we
would know that the measure was not supported by that percentage, since it in fact failed.
Since it was not supported by ten percent, a majority vote could not have taken place, and
the argument would fail.

(E) would strengthen the argument if it were true, but is not necessary. Even if the ten
percent requirement did not guarantee a popular vote defeat (the denial of (E)), the
measure could still be voted down by the majority if the pet lovers succeeded in obtaining
the required ten percent.

• Pre-phrase to cut down on the time wasted on wrong answer choices. With a good
pre-phrase, you’ll zip through bogus answer choices (B), (C), (D) and the torturous
(E). Without a good pre-phrase, you’ll spend time understanding exactly why these
choices are wrong, which might be interesting in rare cases but is generally
hazardous to your score.

16. (C)
We need to weaken the experimenters’ conclusion that fusion was achieved. The
experimenters point to the presence of helium-4, a known by-product of fusion, in an air-
filled chamber. So what? If the amount of helium-4 found in that air was no higher than
that found in ordinary air, answer choice (C), then we would have reason to believe that the
helium-4 gas found in the chamber was there before the experiment. If fusion had been
achieved, then we would expect the level of helium-4 to be higher than the level found in
the air in general. For this reason, answer choice (C) casts doubt on the experimenters’
conclusion.

(A) The last thing (A) does is cast doubt on the argument; we would almost expect to find
other gases in the chamber if fusion was achieved, because we’re told that “one of the by-
products of fusion is helium-4 gas,” implying that there may be others.

(B) Same as (A): this is wholly consistent with the facts in the passage.

(D) The crux of the argument is that helium-4 was found in the chamber, which
supposedly supports the notion that fusion was achieved. What happens to helium over
time is irrelevant—the helium was found in the chamber, and the conclusion was based on
that alone, end of story. Answer choice (D) therefore doesn’t affect this argument one bit.

(E) So what? We’re told nothing about “heat” in the stimulus, so there’s no way this little
tidbit of information can do any damage to the argument. (We’re told that energy is
released, and you may have associated that with heat, but this still doesn’t hurt the
argument—if anything, it means only that (E) is consistent with the passage’s first sentence.)

© K A PL A N 81
LSAT PREP ________________________________________________________________ LSAT Test II Explained: Section IV

• Specialized knowledge is not required on the LSAT. Just like on Science passages
on the Reading Comprehension section, don’t be intimidated by fancy wording or
jargon. You don’t need to know anything about nuclear fusion to answer this
question (thank goodness). You don’t need to know why the experimenters used
“heavy” water, or even what “heavy” water is. But you do need to make the passage
your own and to place the ideas into a simplified general framework that you can
easily access and understand.

• Choices that require additional unsupported assumptions are irrelevant.


Constructing possible worlds in which flawed choices like (A), (B), and (E) might be
relevant is a waste of your time. If you find yourself trying really hard to justify a
choice, it’s probably wrong.

17. (A)
Normally, the worlds of photography and philosophy don’t mix, but every rule has an
exception. The Keyword “Therefore” signals the author’s conclusion, that photographs
cannot provide definite proof. For evidence, the author notes that although photographs
show things that are true in one sense, they cannot show the entire truth. So the author
jumps from the concept of whole truth in the evidence to the concept of definite proof in the
conclusion. Therefore, we are looking for a choice that bridges that gap, and (A) fits the
bill.

(B) is irrelevant. Even if the whole truth remains a mystery, we still don’t know anything
about the relationship of the whole truth to definite proof.

(C) would strengthen the argument if true, but is not necessary. Even if the truthfulness of a
photograph could be known in some sense (the denial of (C)), a photograph would still fail
to provide definite proof if some other obstacle existed, such as the relationship suggested
in (A).

(D) is long and wrong. Answer choice (D) fails to address the issues of definite proof,
brings up for the first time the issue of “corroborative evidence,” and seems to argue for the
usefulness of photographs in the truth-finding process, which conflicts with the author’s
view.

(E) is out of the scope of the argument. The issue is whether photographs may provide
definite proof, not whether something may be definitely proved about the subjects of
photography.

• In Assumption questions, keep your eyes open for a gap between the evidence and
the conclusion; the right answer is always constructed to fill in the missing piece.

82 © K A PL A N
LSAT PREP ________________________________________________________________ LSAT Test II Explained: Section IV

18. (D)
Some household products release toxins, which can be a problem in a well-insulated
house. Fortunately, tests have shown that houseplants remove some household toxins from
the air and thus remove their danger. We’re given the example of a test that showed
houseplants removing formaldehyde from a small, well-insulated house. The scenario in
the question stem is almost exactly like the scenario in that example: a small well-insulated
house containing toxic-releasing products, and the person living there places houseplants
“such as those tested” in the house. So, what would we expect? Just as in the example, if
there’s any formaldehyde in the air, we’d expect the houseplants to reduce it. That’s (D).

(A) goes much too far. First, we don’t know that the plants will remove all the toxins from
the air, so ventilation still might be advisable to get rid of toxins. Secondly, there might be
lots of other reasons to ventilate the house.

(B) wouldn’t be expected; we know houseplants cut down on some toxins, including
formaldehyde, so it would be a surprise if the plants had absolutely no effect on the
concentration of toxins.

(C) goes too far in saying that the air supply will be “safe”—even in the example, when
there were 20 large plants in the house, we know only that formaldehyde was eliminated, not
any other toxins. Moreover, although the house is well-insulated, it mightn’t be warm;
maybe it’s poorly heated.

(E) includes a scope shift: the quantities released are different from the quantities that
houseplants can remove from the air. A subtle, yet major, difference. We have no idea how,
or even if, houseplants affect the quantities of toxins released. We know only what effect
some houseplants may have after such toxins are in the air.

• Don’t let a long, complicated-looking two-part stem like this one intimidate you.
Often, as in this case, such stems merely mirror, and therefore preview for you, the
contents of the stimulus.

19. (D)
Some household products release dangerous toxins in the air; these toxins are not a
problem in a well ventilated house but can be a problem in a poorly ventilated house. In
poorly ventilated houses, however, it’s been found that houseplants can do a good job of
removing toxins from the air and thus compensate for the lack of ventilation. (D) expresses
the point of this passage best: keeping houseplants can compensate for some of the bad
effects (i.e. toxin-poisoned air) of poor ventilation.

© K A PL A N 83
LSAT PREP ________________________________________________________________ LSAT Test II Explained: Section IV

(A) The passage never explicitly says that benzene is one of the toxins that can be removed
by houseplants; benzene is mentioned only in the beginning as an example of a toxin. The
rest of the passage focuses exclusively on formaldehyde. Thus, the passage is certainly not
structured to lead to the conclusion here.

(B) The author of this passage has no interest in what doesn’t release toxins; he’s interested
in how to deal with toxins that are released. Moreover, we don’t know that non-synthetic
products don’t release toxins; maybe some of the toxin-releasing wall paneling mentioned
by the author is non-synthetic.

(C) is a distortion. The author said that houseplants can counter some of the negative
aspects of living in a poorly ventilated, well-insulated house, not that houseplants can
counter the negative aspects of living in a poorly insulated house; the author certainly
didn’t say that houseplants conserve heat.

(E) isn’t supported by the passage. The author never directly compared the effectiveness of
good ventilation to that of houseplants in controlling toxins. Indeed, if you had to make a
guess, since good ventilation is described as removing the “problem” of toxins, whereas
houseplants are only described as removing “some” household toxins, it looks as if the
edge in effectiveness would go to ventilation.

• When you’re looking for the main point of the passage (and essentially that’s what
we’re asked for), ask yourself as you read the passage: why is the author telling me
this? What is this supposed to make me believe? Actually, it’s a good idea to think
that way on all question types—you can only evaluate the argument if you
understand how it works and what it’s supposed to prove.

• Both “benzene” and “synthetic” material are mentioned just once at the beginning of
the passage. Such minor details can never provide the main point, which is
supposed to be a summation of the whole argument’s purpose.

20. (D)
We can symbolize the first sentence: “all X (normal full-term babies) are Y (born with
certain reflexes that disappear by the age of two months).” Then the argument runs, “NOT-
Y (this baby’s reflexes have not disappeared by the end of two months), therefore NOT-X
(this baby isn’t a full-term baby).” Simple enough: all X are Y; NOT-Y, therefore NOT-X.
The parallel is (D): all X (“Opossums,” or “All opossums”) are Y (possess abdominal
pouches). NOT-Y (this animal does not possess an abdominal pouch), therefore NOT-X
(this animal isn’t an opossum).

(A) runs: all X (Carbon dioxide or “All carbon dioxide”) is Y (turns limewater milky).
NOT-X (this gas is NOT carbon dioxide; i.e. it’s oxygen), therefore NOT-Y (it won’t turn
limewater milky). Invalid and not parallel.

84 © K A PL A N
LSAT PREP ________________________________________________________________ LSAT Test II Explained: Section IV

(B) runs: all X (All apes) are Y (are unable to talk). X (Suzy is an ape), therefore Y (Suzy
can’t talk). Again, not parallel.

(C) is a casual argument that doesn’t even adapt well to symbolization. We’re never told
that Henry is human—we have to assume that. Moreover, the conclusion introduces the
concept of “normalcy” in general, whereas in the stimulus normalcy is used in regard to a
very specific respect (a normal full-term baby).

(E) begins talking about “some” trees instead of “trees” or “all trees,” so right away you
know it doesn’t parallel the stimulus. Moreover, (E) is a casual argument that fudges its
terms, going from “sheds its leaves annually” to “has not shed its leaves.” When? So far?
Ever? There’s no such uncertainty in the stimulus.

• As always on formal logic questions the chief problem is one of translation. Not only
do you need to recognize that a categorical statement like “normal full-term babies
are all born with certain instinctive reflexes that disappear after two months”
translates into “All X are Y,” you also need to recognize that, for the purposes of the
argument, “these reflexes are present after three months” means essentially “these
reflexes have not disappeared at the end of two months,” or “NOT-Y.”

21. (E)
According to the last sentence, if the human auditory system is an imperfect receptor of
sounds then it is impossible for every possible permutation of a spoken language’s basic
language sounds to be an understandable word. But the previous sentence said that if a
spoken language is completely efficient, then it must be true that every permutation of its
basic language sounds can be an understandable word. When we combine those two
thoughts we get (E): if the human auditory system is an imperfect receptor, then not every
permutation of its basic language sounds is an understandable word. This triggers the
contrapositive of the second sentence, allowing us to conclude that no language is
completely efficient.

(A) The stimulus gave no grounds for associating efficiency with usefulness or
redundancy with reliability; instead, the stimulus said that together efficiency and
redundancy could help bring about both usefulness and reliability.

(B) The stimulus never said whether or not redundancy is possible in a spoken language;
in fact, the stimulus never mentioned redundancy after the first sentence.

(C) The stimulus never even explained what “redundancy” in a language means; certainly
we have no idea whether “complete redundancy” rules out usefulness in a language.

(D) assumes that the imperfection of the human auditory system is the only obstacle
preventing every permutation of language sounds from being an understandable word.
The stimulus, however, merely said that this imperfection was an obstacle, not that it was
the only obstacle.

© K A PL A N 85
LSAT PREP ________________________________________________________________ LSAT Test II Explained: Section IV

• The stimulus drops the theme of redundancy after the first sentence, never even
bothering to define it. The passage is thus fragmented—not a smooth logical
argument—and it should be no surprise to find that the inference is based only on
part of the passage.

• If you read actively—really paying attention as you read—you’re more likely to


notice when the idea of “every possible permutation of a language’s sounds being an
understandable word” comes up the second time, and to ask yourself, “What did
they say just about that? Didn’t they say that’s what perfect efficiency relies on?”
You’ll have the answer then, before you even look at the choices.

• (D) makes a common logical blunder, usually called “Denying the Antecedent” (the
antecedent is the “if” clause). Denying the antecedent means taking “If X, then Y”
and inferring “If NOT-X, then NOT-Y.” In terms of the contrapositive, it means that
we’ve negated the terms of the original if-then statement, but have forgotten to
switch them around.

22. (D)
First, the stimulus takes the statement “All intelligent people are nearsighted” (“All X are
Y” or “If X, then Y”) and tries to conclude something based on the consequent—i.e., on the
Y term. So the first mistake the stimulus makes is to argue along the lines “All intelligent
people are nearsighted; I am nearsighted, therefore I must be intelligent.” But the stimulus
doesn’t stop there: it intensifies both terms, arguing that since I am very nearsighted, I must
be very intelligent (a genius, in fact). There’s no grounds for connecting degree of
nearsightedness to degree of intelligence, so that’s a second flaw. (D) shows the same
pattern. Based on the statement “All tall people are happy” or “All X are Y,” (D) infers not
simply that “Since John is happy, he must be tall” (or “Y, therefore X”) but goes the extra
mile and concludes “Since John is extremely happy, he must be extremely tall.”

(A) takes the evidence, “All intelligent people are nearsighted” (All X are Y), and goes on
to infer something from the negation of the consequent or the Y term: “I have perfect vision”
(i.e. “I am NOT nearsighted” or NOT-Y), therefore I must be stupid” (i.e. “I am NOT
intelligent” or NOT-X). Thus, although (A) does include the same intensification we find in
the stimulus, the structure of the argument is different.

(B) follows the stimulus in reasoning “All X are Y; Y, therefore X” but misses the second
flaw, that of intensifying the terms.

(C) is a mess; there’s nothing connecting the idea of the number of legs an animal has to its
size, nothing relating pigs to spiders, and, in fact, nothing remotely similar to the stimulus.

(E) simply runs “All X are Y (All geniuses are very nearsighted); X (I am a genius),
therefore Y (I must be very nearsighted). That’s valid and not parallel.

86 © K A PL A N
LSAT PREP ________________________________________________________________ LSAT Test II Explained: Section IV

• Use the keywords to help you separate evidence from conclusion in the choices. In
(D), the evidence comes in the second sentence, as signaled by “because.” In (E), the
second sentence is the conclusion, but the conclusion follows in inverse order: “I
must be very nearsighted since I am a genius” inverts to “I am a genius, therefore I
must be very nearsighted.”

• The stimulus’ initial logical flaw, taking “All X are Y” or “If X, then Y” and inferring
“If Y, then X,” is called “Affirming the Consequent.” Along with “Denying the
Antecedent,” (see Q. 21) it’s one of the most common logical flaws on the LSAT.

23. (C)
We’ve got two basic rules to start with: 1) All headache pills (including Danaxil) can stop a
headache and 2) No headache pill can stop a headache faster than Danaxil. Then we’re
given a situation to which the rules must be applied: Evelyn takes Danaxil for her
headache, whereas Jane takes a competing pill for her headache. What can we conclude?
Well, we know that both headaches will be relieved (Rule 1) and we also know that Jane’s
headache will not be relieved faster than Evelyn’s (Rule 2). As (C) says, this boils down to
the fact that Evelyn’s headache will be relieved at least as quickly as Jane’s.

(A) isn’t true; since “all headache pills can stop your headache,” Jane’s headache will be
relieved.

(B) The advertisement never said that Danaxil relieves headaches more quickly than any
other headache pill, only that no other pill works faster than Danaxil. So we only know that
Danaxil works at least as fast as any other pill.

(D) There’s no reason to think that the two headache remedies will finish in a dead heat; it’s
possible Danaxil will work faster.

(E) We have no grounds for inferring that Jane will take Danaxil; her headache will be
relieved (however slowly) by the pills she’s taken, after which she won’t need any more
headache pills until such time as another headache comes along. We don’t know what
she’ll take then.

• This question is just a matter of reading carefully and recognizing the limits of the
claim “no headache pill stops pain more quickly.”

• Remember the first principle of Section Management—all questions count the same,
and you should not get bogged down on tough ones along the way that may make
you miss out on easy questions. Although this is a fairly easy question, it’s number
23. It would be a pity if this were Test Day and you never even got to look at this
question because you spent too much time wrestling with a really tough question
earlier in the section.

© K A PL A N 87
LSAT PREP ________________________________________________________________ LSAT Test II Explained: Section IV

24. (D)
Reading the stem first tells us this is a method of argument question and that we should
attack the stimulus looking for how the evidence relates to the conclusion. Back in 1970,
automakers claimed that meeting the environmental standards proposed at the time was
too costly and unnecessary. However, a technological development allowed the
automakers to meet the standard while keeping costs in line. Now, new, stricter standards
are being considered, and the automakers are making almost the same argument they did
in 1970. The author implies that since automakers are currently making factual claims that
are extremely similar to the dubious claims that were made in 1970, we should be skeptical
of those current claims. In other words, the author compares their current position with a
position held in the past, which is choice (D).

(A) No contradiction here. It could be that meeting the old standards was feasible but that
meeting the new standards is not. There is no guarantee that technology will come to the
rescue again.

(B) What facts?

(C) The author takes issue with the claims of fact made by the automakers, not with their
reasoning. For (C) to be the right answer, the author would have to accept the factual
claims made by the automakers but argue that they do not support the automakers’
conclusion. That doesn’t happen here.

(E) What evidence? It simply doesn’t happen, so cross it out.

• In Method of Argument questions, it’s often helpful to try to imagine what the
stimulus would have to sound like if the choice you’re analyzing were correct. As
noted in (C) above, the stimulus would have to be radically different for this choice
to describe the author’s method. Recognizing this allows us to eliminate it with
confidence.

88 © K A PL A N
LSAT PREP ________________________________________________________________ LSAT Test II Explained: Section IV

25. (A)
Finally, we need to see what would support the automakers’ position. As explained above,
the automakers’ current claims about unreasonable costs could be true even though
similar claims were false in another context. This time things could be different, since
technology might not be able to keep pace in reducing emissions efficiently. If stricter
standards make it harder for automakers to provide the required technology (A), then we
would have reason to believe that the current situation differs from the one in 1970, and the
automakers’ current position would be strengthened.

(B) is out of the scope. The fuel efficiency of cars is not an issue in the argument.

(C) could only weaken the argument by suggesting that the new standards will not be
overly burdensome to automakers.

(D) would weaken the argument by supporting the view that stricter emission standards
are necessary, which of course is the opposite of the automakers’ view.

(E) is irrelevant. The issue is whether the new standards are feasible and necessary, not the
general motivations and practices of automakers.

• Don’t let your personal opinions interfere with how you handle the LSAT. On Test
Day, you might well have to answer a question based on an issue on which you have
a personal opinion. In this case, you might have an opinion on the issue of auto
emissions, but you still need to be objective. Always base your answer on the
argument as it appears in the stimulus.

© K A PL A N 89
I.N. LL3123 Rev.A Printed in the USA

You might also like